Download as pdf or txt
Download as pdf or txt
You are on page 1of 150

PREPARATION FOR THE

PE CIVIL EXAM

PE GEOTECHNICAL & MATERIALS RELATED TOPICS


REFRESHER NOTES
Refresher Class Geotechnical

Table of Contents

1.0 Phase Relationships .......................................................................................... 1


1.1 Phase Diagram ............................................................................................................ 1
1.2 Basic Soil Property Definitions ..................................................................................... 1
1.3 Phase Relationships .................................................................................................... 2
EXAMPLE 1.0 ............................................................................................................. 4
1.4 Relative Density........................................................................................................... 5
EXAMPLE 1.1 ............................................................................................................. 6
2.0 Index Properties & Soil Classification .............................................................. 7
2.1 Grain Size Distribution ................................................................................................. 7
EXAMPLE 2.0 ............................................................................................................. 9
EXAMPLE 2.1 .......................................................................................................... 10
2.2 Atterberg Limits ......................................................................................................... 11
2.3 AASHTO Classification System ................................................................................. 12
EXAMPLE 2.2 ........................................................................................................... 14
2.4 Unified Soil Classification System (USCS) ................................................................. 15
EXAMPLE 2.3 ........................................................................................................... 19
2.5 USDA Soil Classification System ............................................................................... 15
EXAMPLE 2.4 ........................................................................................................... 22
3.0 Soil Sampling and In-Situ Testing Methods ................................................... 23
3.1 Undisturbed Sampling ............................................................................................... 23
3.2 Disturbed Sampling ................................................................................................... 23
3.3 Rock Quality ............................................................................................................. 24
3.4 Common In-Situ Tests ............................................................................................... 25
3.5 Standard Penetration Test (SPT) ............................................................................... 26
EXAMPLE 3.0 ........................................................................................................... 27
EXAMPLE 3.1 ........................................................................................................... 29
3.6 Common Laboratory Tes ........................................................................................... 30
4.0 Compaction and Pavement Design Parameters ............................................ 32
4.1 Proctor Laboratory Tests ........................................................................................... 33
EXAMPLE 4.0 ........................................................................................................... 38
4.2 Field Density Tests .................................................................................................... 36
4.3 Relative Compaction.................................................................................................. 38
EXAMPLE 4.1 ........................................................................................................... 38
4.4 California Bearing Ratio ............................................................................................. 39
4.5 Subgrade Modulus .................................................................................................... 40
4.6 Resilient Modulus ...................................................................................................... 41
4.7 Expansive Soils ......................................................................................................... 42
4.8 Frost Susceptibility ................................................................................................... 42
5.0 Permeability & Seepage .................................................................................. 43
5.1 Darcy’s Law and Bernoulli’s Equation ........................................................................ 43
EXAMPLE 5.0 ........................................................................................................... 48
5.2 Seepage Velocity....................................................................................................... 49
5.3 Coefficient of Permeability Laboratory Tests .............................................................. 49
EXAMPLE 5.1 ........................................................................................................... 50
EXAMPLE 5.2 ........................................................................................................... 52
5.4 Flow Nets .................................................................................................................. 53
EXAMPLE 5.3 ........................................................................................................... 54
6.0 Vertical Stresses and Pore Water Pressure ................................................... 55

i
Refresher Class Geotechnical

6.1 Total Vertical Stress .................................................................................................. 55


6.2 Pore Water Pressure ................................................................................................. 56
EXAMPLE 6.0 ........................................................................................................... 57
6.3 Effective Vertical Stress ............................................................................................. 57
EXAMPLE 6.1 ........................................................................................................... 58
7.0 Pressure Distribution ....................................................................................... 59
7.1 Point Loads ............................................................................................................... 59
7.2 Uniformly Loaded Areas ............................................................................................ 59
EXAMPLE 7.0 ........................................................................................................... 68
EXAMPLE 7.1 ........................................................................................................... 68
8.0 Consolidation of Clay Soils ............................................................................. 70
8.1 Overview of Settlement in Clay Soils ......................................................................... 70
8.2 Stress History of Clay Soils........................................................................................ 71
8.3 Consolidation of NC Clays ......................................................................................... 72
EXAMPLE 8.0 ........................................................................................................... 73
8.4 Consolidation of OC Clay .......................................................................................... 74
8.5 Empirical Relationships of Cc and Cr ......................................................................... 76
8.6 Rate of Consolidation ................................................................................................ 76
EXAMPLE 8.1 ........................................................................................................... 78
9.0 Shallow Spread Foundations .......................................................................... 79
9.1 Shear Strength Parameters ...................................................................................... 79
9.2 Types of Foundations ................................................................................................ 80
9.3 Bearing Pressure of Shallow Spread Foundations ..................................................... 83
9.4 General Bearing Capacity Theory .............................................................................. 84
EXAMPLE 9.0 ........................................................................................................... 88
9.5 Bearing Capacity in Clay ........................................................................................... 89
EXAMPLE 9.1 ........................................................................................................... 89
9.6 Bearing Capacity in Sand .......................................................................................... 90
EXAMPLE 9.2 ........................................................................................................... 90
9.7 Effect of Groundwater Table on Bearing Capacity ..................................................... 91
EXAMPLE 9.3 ........................................................................................................... 92
9.8 Eccentric Loads on Rectangular Shallow Spread Footings ........................................ 93
EXAMPLE 9.4 ........................................................................................................... 95
10.0 Lateral Earth Pressure and Earth Retention Structures ............................... 97
10.1 Earth Pressure Introduction ....................................................................................... 97
10.1.1 At-Rest Earth Pressure ................................................................................... 98
10.1.2 Active Earth Pressure ..................................................................................... 98
10.1.3 Passive Earth Pressure................................................................................... 99
10.2 Rankine’s Earth Pressure Theory ............................................................................ 100
10.3 General Earth Pressure Diagrams ........................................................................... 101
10.4 Coulomb’s Earth Pressure Theory ........................................................................... 103
10.5 Rigid Retaining Walls............................................................................................... 104
EXAMPLE 10.0 ....................................................................................................... 106
10.6 Stability of Rigid Retaining Walls ............................................................................. 107
EXAMPLE 10.1 ....................................................................................................... 110
10.7 Equivalent Fluid Pressure ........................................................................................ 107
EXAMPLE 10.2 ....................................................................................................... 110

ii
Refresher Class Geotechnical

11.0 Slope Stability and Braced Excavations ....................................................... 113


11.1 Slope Stability .......................................................................................................... 120
11.1.1 Taylor’s Slope Stability Chart .................................................................... 120
EXAMPLE 11.0 ....................................................................................................... 121
11.1.2 Method of Slice ......................................................................................... 116
11.2 Braced Excavations ................................................................................................. 118
11.2.1 Pressure Diagrams for Cross-Braced Excavation .............................................. 120
EXAMPLE 11.1 ............................................................................................... 122
11.2.2 Analysis of Cross-Braced Excavation ............................................................... 120
EXAMPLE 11.2 ...................................................................................................... 122

APPENDIX A ................................................................................................................... I
APPENDIX B ................................................................................................................. VI
APPENDIX C ................................................................................................................. XI

Recommended Supplelemental References for the


Geotechnical Portion of the PE Exam

1. Lindeburg, M. R. (2018), Civil Engineering Reference Manual for PE Exam, 16 th


ed, Professional Publications, Inc., Belmont, CA.
2. Das, B. M. (2007), Principles of Foundation Engineering, 6th ed, Nelson –
Thompson Canada Limited., Toronto, Ontario
3. Coduto D. P. (1999), Goetechnical Engineering: Principles and Practices, Prentice
Hall, Upper Saddle River, NJ.
4. Duncan, J.M., Buchignani, A.L., and De Wet, M. (1987), Engineering Manual for
Slope Stability Studies, Charles E. Via, Jr. Department of Civil Engineerring,
Virginia Polytechnic Institute and State University, Blackburg, VA.
5. Liu, C. and Evett, J. B. (2004), Soils and Foundations, 6th ed, Pearson Education,
Inc., Upper Saddle River, NJ.
6. McCarthy, D.F. (2007), Essentials of Soil Mechanics and Foundations: Basic
Geotechnics, 7th ed, Pearson Education, Inc., Upper Saddle River, NJ.
7. NAVFAC (1982a), Soil Mechanics: Naval Facilities Engineering Command Design
Manual, NAVFAC DM-7.1, U.S. Department of the Navy, Alexandria, Virginia.
8. NAVFAC (1982b), Foundations and Earth Structures: Naval Facilities Engineering
Command Design Manual, NAVFAC DM-7.2, U.S. Department of the Navy,
Alexandria, Virginia.

FIGURES
Figure 1.0 Phase Diagram…………………………………………………………………….1

iii
Refresher Class Geotechnical

Figure 2.1 Sample Grain Size Distribution Curves...……………………………………….8


Figure 2.2 Consistency of Fine-Grained Soils........………………………………………..11
Figure 2.3 AASHTO Graphical Chart......…….........……………………………………….13
Figure 2.4 USCS Coarse-Grained Classification Flow Chart...…………………………..17
Figure 2.5 USCS Fine-Grained Classification Flow Chart…....…………………………..18
Figure 2.6 Casagrande’s Plasticity Chart....………………….....………………………….18
Figure 2.7 USDA Soil Texture Classification Chart…….….…...………………………….21
Figure 3.0 Hollow-Stem Sampling.………………….…….……...………………………….24
Figure 3.1 Sample Boring Log..….………………….…….……...………………………….28
Figure 4.0 Moisture Unit Weight Relations..……….…….……...………………………….32
Figure 4.1 Application of compaction equipment by soil type…....……………………….33
Figure 4.2 A typical compaction test protor curve...…….……...………………………….34
Figure 4.3 Rubber Ballon – Sand Cone Test Method….……...…………………………..36
Figure 4.4 Typical Schematic of a Nuclear Density Gauge…...…………………………..37
Figure 4.5 Correlations of Subgrade Modulus to CBR..….…...…………………………...38
Figure 5.0 Hydraulic Gradient as Head Loss between Two Points..…..………………. ..43
Figure 5.1 No Flow………………..……………………………………..……...……………..45
Figure 5.2 Downward Flow….……………………………..……...………………………….46
Figure 5.3 Upward Flow…..………………………………..……...………………………….45
Figure 5.4 Constant Head Apparatus…..…..………………………………………………..49
Figure 5.5 Falling Head Apparatus…...…………………..……...…………………………..51
Figure 6.0 Example for Total Stress..………………………………………………………..55
Figure 6.1 Example s of pressure head for hydrostatic (left) and
seepage (right) cases……………………………………………….…………….56
Figure 7.0 Boussinesq stress contours for uniformly loaded circular areas……………..61
Figure 7.1 Inflence factors for finding vertical stress uder the corner of
a recatngualr laoded area......…….........……………………………………….64
Figure 7.2 Inflence factors for finding vertical stress uder the corner of
a recatngualr laoded area (Boussinesq Cas)………………………………….65
Figure 7.3 Boussinesq stress contours for uniformly loaded infinitely long
and square footings…………………………...………………………………….66
Figure 7.4 Approximate method for rectangular area….…….....………………………….67
Figure 8.0 Three phases of settlement in fine-grained soils…...………………………….70
Figure 8.1 Typical 1-D Consolidation Test……………….……...………………………….71

iv
Refresher Class Geotechnical

Figure 8.2 1-D Consolidation Test of NC Clay…….…….……...………………………..72


Figure 8.3 1-D Consolidation Tests of OC Clays (Case I and Case II)………………..74
Figure 8.4 Two-way and One-way Drainage Paths……………....……………………….77
Figure 9.0 Shallow spread footing dimensions…...…….……...………………………….80
Figure 9.1 Eccentrically loaded foundations…………….……...…………………………..93
Figure 10.0 Nature of Lateral earth pressure on a retaining wall..……………………….97
Figure 10.1 Gravity retaining walls (a) Rankine and (b) Coulomb……………………....105
Figure 10.2 Cantilever retaining wall (Rankine)……………………...…..……………….105
Figure 11.0 Taylor’s chart modified by Janbu for saturated clay slopes.....……………114
Figure 11.1 Slope with an arbitrary failure surface divided into slices and
forces acting on an individual slice…..……............…………………………116
Figure 11.2 Slope with arbitrary failure surface for nonuniform soil………………….....117
Figure 11.3 Typical slope for cohesionless soil (sand)..………………………………….117
Figure 11.4 Support system for walled-excavation……..……...…………………………118
Figure 11.5 Apparent pressure diagrams on braced vertical sheeting.…………………120

TABLES
Table 1.0 Phase Relationships……………………………………………………………….2
Table 2.0 US Standard Sieve Sizes………………………………………………………….7
Table 2.1 AASHTO Classification Table..………………………………………………….13
Table 3.0 Descriptive Terms of Rock Quality...……………………………………………24
Table 3.1 Summary of Common In-Situ Tests for Soils.………………………………….25
Table 3.2 Strength of Cohesive Soils.……………………………………………………....27
Table 3.3 Relative Density of Granular Soils……………………………………………….27
Table 3.4 Summary of Common Laboratory Tests for Soils….…………………………..30
Table 4.0 Typical Resilient Modulus Values.………………….…………………………....41
Table 7.0 Influence factors for finding vertical stress under a circular loaded area…….60
Table 7.1 Influence factors for finding vertical stress uder the
corner of a rectangular loaded area……………………………………………..62
Table 8.0 Approximate Time Factors………………………………………………………..77
Table 9.0 Foundation types correlated with soil conditions……………………………….80
Table 9.1 Terzaghi’s Bearing capacity factors.……………………………………………..86
Table 9.2 Shape factors for various B/L values…………………………………………….87
Table 10.0 Typical range of lateral earth pressure coefficients.…………………………. 97

v
Refresher Class Geotechnical

Table 11.0 Factors in selected of support system for Deep Excavation.…….………….119


Table 11.1 Computation of Pmax by soil type...…………………………………………….. 120

vi
Refresher Class Geotechnical

1.0 Phase Relationships

1.1 Phase Diagram

Figure 1.0: Phase Diagram

Total Volume = V a + V w + Vs Total Weight = Ww + Ws


= Vv + Vs

1.2 Basic Soil Property Definitions

Weight of Water Ww
Moisture Content, w   100%
Weight of Solids Ws
Volume of Water Vw
Degree of Saturation, S   100%
Volume of Voids Vv

Total Weight Wt
Total Unit Weight, t  
Total Volume Vt
Weight of Solids Ws
Dry Unit Weight, d  
Total Volume Vt
Volume of Voids Vv
Void Ratio, e 
Volume of Solids Vs
Volume of Voids Vv
Porosity, n 
Total Volume Vt
Unit Weight of Solids  solids
Specific Gravity, Gs  
Unit Weight of Water w

1
Refresher Class Geotechnical

1.3 Phase Relationships

 By combining the basic definitions, we can derive a new series of


phase relationship equations.

Table 1.0: Phase Relationships


(Compare to CERM Table 35.7)

WEIGHT

W Ws  Ww W s (1  w ) V

W WwGs VG s  w
Ws W  Ww V d Vs Gs w VG s  w (1  n )
1 w Se 1 e

W s Se
Ww W  Ws wWs Vv S  w Vw w
Gs

VOLUME
W Ws Vs Vv (1  e) Vv
V Vs  Vw  Va Vs (1  e)
 d 1 n e n

Ws V Vv Vv (1  n)
Vs V  Vw  Va V (1  n)
Gs w 1 e e n

Ww eSV nSV s
Vw V  Vs  Va nSV eSVs SVv
w 1 e 1 n

Ws eV nVs Vw
Vv V  Vs V nV eVs
Gs w 1 e 1 n S

eV (1  S ) nVs (1  S )
Va V  Vs  Vw nV (1  S ) eVs (1  S ) Vv (1  S )
1 e 1 n

2
Refresher Class Geotechnical

UNIT WEIGHT
 w (1  w)
W  w (Gs  eS ) Gs w
  d (1  w ) w 1 (1  w)
V 1 e  1 e
S Gs
Gs w
Ws  Gs  w
d G s  w (1  n ) wG s
V 1 w 1 e 1
S
 w (1  w)
sat W Ws  Vv  w  e   w (Gs  e)
 d   w  1
(S=100%) V V 1  e  w 1 e
Gs

b = ’  Gs  e 
 sat   w  d   w 1  n  w  1
(buoyant)  1 e 

Ww Ws  d (1  e) d
w
Vw Vs Gs Gs Gs (1  n)
Ws Ww
solid  wGs
Vs VwG s

WEIGHT-VOLUME RELATIONSHIPS
Ww W   1  eS
w [x 100%] 1 1 S  w  
Ws Ws d 
 d G s
Gs

w
Vw Ww wGs
S [x 100%] w 1
Vv  wVv e 
 d Gs

Vv V WwGs Gs w n wGs


e 1 1
Vs Vs Ws S d 1 n S

Vv Vs Ws d e
n 1 1 1
V V VGs  w Gs w e 1

 solid Ws  d (1  e) eS
Gs w Vs w w w

3
Refresher Class Geotechnical

Notes: 1. These relationships can also be written in terms of mass and/or


density by substituting weight (W) with mass (M) and unit weight
() with density ().

Ws Ms
ex: Gs  or Gs 
Vs  w Vs  w

Where the relationship of weight to mass is as follows:

W  Mg where g = 32.2 ft/s2 (English) = 9.81 m/s2 (SI)

2. The typical unit weight of water is as follows:

w = 62.4 lbf/ft3 (English) = 9.81 kN/m3 (SI)

3. Reference CERM (2018) Chapter 35.5, Das (2007) Chapter 1.4,


and/or McCarthy (2007) Ch 3.

EXAMPLE 1.0
A soil has a volume of 0.3 ft3 and weighs 36 lbs. The specific gravity (Gs)
of the soil sample is 2.67 and the moisture content (w) is 18%. Determine
the moist unit weight (), dry unit weight (d), void ratio (e), porosity (n),
and degree of saturation (S):

Solution:
Use Table 1.0 to solve directly for required parameters. Fill in the blank
boxes with an equation that can be used to solve for each parameter:

  120 lb
d   101.7 lb
ft 3 ft 3

e  0.64 n  0.39

S  75%

(Answers given in Appendix A; Alternative Solution in Appendix B)

4
Refresher Class Geotechnical

1.4 Relative Density

 Special weight-volume relationship used in sands and gravels (not


applicable to fine-grained soils).

 Expressed in terms of void ratio or dry unit weight as determined by


lab test:

emax  e
Dr   100%  e  emax  Dr (emax  emin )
emax  emin

or

  d  ( d ) min  ( d ) max 
Dr      100%
 ( d ) max  ( d ) min   d 

( d ) min
 d 
 Dr 
1    ( d ) max  ( d ) min 
 ( d ) max 

where: Dr = Relative Density


emax = Maximum void ratio
emin = Minimum void ratio
e = In-situ void ratio
(d)max = Maximum dry unit weight
(d)min = Minimum dry unit weight
d = In-situ dry unit weight

 See Table 3.3 for correlations of relative density to Standard


Penetration Test N-values and granular shear strength parameter
(internal angle of friction, ).

 Reference CERM (2018) Chapter 35.5, Das (2007) Ch 1.5, and/or


McCarthy (2007) Ch 4.

5
Refresher Class Geotechnical

EXAMPLE 1.1
For a given sand sample labortory tests show emax = 0.41, emin = 0.25, and
Gs = 2.72. An in-situ Standard Penetration Test (SPT) indicates that the
relative denisity of the sand is approximately 65%. Estimate the in-situ dry
unit weight of the sand.

Solution:
Use Table 1.0 to pick an equation for dry unit weight that matches
information that is known or can be solved for:

 Known: emax, emin, Dr  Solve for ein-situ


 Known: Gs

Gs w
 Use  d 
1 e

Use the relative density equation to solve for the in-situ void ratio:

e  emax  Dr (emax  emin )


e  0.41  0.65(0.41  0.25)  0.31

Solve for the in-situ dry unit weight:

Gs w 2.72(62.4 pcf )


d    130 pcf
1 e 1  0.31

6
Refresher Class Geotechnical

2.0 Index Properties & Soil Classification

2.1 Grain Size Distribution


 Sieve Analysis used to obtain the grain size distribution of coarse-
grained soils (sands and gravels) larger than 0.075 mm (retained
above No. 200 Sieve)

 Hydrometer Analysis used to obtain the grain size distribution of


fine-grained soils (finer sands, silts and clays) smaller than 0.150 mm
(passing No 100 Sieve)

 Reference CERM (2018) Chapter 35.1, Das (2007) Chapter 1.2,


and/or McCarthy (2007) Ch 4.
Table 2.0: US Standard Sieve Sizes
(CERM Table 35.2)
Sieve Size Sieve Opening (mm)
4 in. 100
3 in. 75
2 in 50
1 ½ in. 37.5
1 in. 25
¾ in. 19
½ in. 12.5
3/ in. 9.5
8

No. 4 4.75
No. 8 2.36
No. 10 2.00
No.16 1.18
No. 20 0.850
No. 30 0.600
No. 40 0.425
N0. 50 0.300
No. 60 0.250
No. 70 0.212
No.100 0.150
No.140 0.106
Figure 2.0: Sieve Analysis
No. 200 Coduto, 1999)
(Source: 0.075

7
Refresher Class Geotechnical

Figure 2.1: Sample Grain Size Distribution Curves


(CERM Figure 35.3)

 Grain Size Distribution Curve is a plot of “percent finer” vs. “particle


diameter” in mm on a log scale.

 Distribution shape indices, coefficient of uniformity, Cu and coefficient


of curvature, Cc indicate the general shape of the curve. NOTE: Cc is
also known as Cz (coef. of gradation) in some texts, including the
CERM.
D ( D )2
Cu  60 and Cc (aka C z )  30
D10 D60 D10

 Dn is the particle size (diameter in mm) at which “n” percent of the


particles are finer.

 The “mean particle size” (D50) is the particle diameter at which 50%
of the particles are finer.

 The “effective particle size” (D10) is the particle size at which 10% of
the particles are finer.

8
Refresher Class Geotechnical

EXAMPLE 2.0
Determine the coefficient of uniformity and the coefficient of curvature of
the “gap-graded” and “well-graded” soils shown in Figure 2.1 on the
previous page.

Solution:

“Gap-Graded” Soil:
D60  6.0 mm, D30  0.3 mm, D10  0.08 mm

D60 6.0 mm D30 2 (0.3 mm) 2


Cu    75 Cc    0.188
D10 0.08 mm D60 D10 (6.0 mm)(0.08 mm)

“Well-Graded” Soil:
D60  1.0 mm, D30  0.15 mm, D10  0.02 mm

D60 1.0 mm D30 2 (0.15 mm) 2


Cu    50 Cc    1.13
D10 0.02 mm D60 D10 (1.0 mm)(0.02 mm)

9
Refresher Class Geotechnical

EXAMPLE 2.1
The results of a sieve analysis are as follows. Determine the mean grain
size, effective grain size, coefficient of uniformity and coefficient of
gradation.

Sieve Size Percent


Finer
No. 4 100
No. 10 72
No. 20 60
No. 30 50
No. 40 44
No. 60 30
No. 200 10

Solution:

*Use Table 2.0 to complete the following:

Sieve Size Sieve Percent


Opening (mm) Finer
No. 20 60 D60
No. 30 50 D50
No. 60 30 D30
No. 200 10 D10

The mean grain size = D50 = _______________

The effective grain size = D10 = ______________

Calculate the coefficient of uniformity:


D60   
Cu    11.33
D10  
Calculate the coefficient of curvature:
 
2
D302
Cc    0.98
D60 D10   
(Answers given in Appendix A)

10
Refresher Class Geotechnical

2.2 Atterberg Limits

Liquid State
(Liquid Behaviour)
Liquid Limit (LL)
Plastic State PI = LL-PL
(Add Water - Remold)
Plastic Limit (PL)
Semi-solid State
(Add water - Remold)
Solid State
(Add water - Remold)

Figure 2.2: Consistency of Fine-Grained Soils

 The Atterberg Limits tests are designed to determine the plasticity of


finer grained soils. These tests are performed on the fraction less
than 0.425 mm (passing the No. 40 Sieve).

 The liquid limit (LL) is the moisture content at which the soil sample
passes from the plastic state to the liquid state.

 The plastic limit (PL) is the moisture content at which the soil sample
passes from the semisolid to the plastic state.

 The plasticity index (PI) is the difference in moisture content from the
threshold of the plastic to liquid state to the threshold of the semisolid
to plastic state.

PI = LL – PL

 Reference CERM (2018) Chapter 35.14, Das (2007) Chapter 1.6,


and/or McCarthy (2007) Ch 4.

11
Refresher Class Geotechnical

2.3 AASHTO Classification System

 Particle Size Definitions

Gravel: 75 mm (3 in.) to 2.0 mm (No. 10)


Sand: 2.0 mm (No. 10) to 0.075 mm (No. 200)
Silt: 0.075 mm (No. 200) to 0.002 mm
Clay: 0.002 mm to 0.001 mm

 Classification Procedures

1. Identify the percent passing the No.10, No. 40, and No. 200
sieves. Identify the LL and PI of the fraction passing the No. 40
sieve.

2. Use AASHTO Classification Table (Table 2.1). Move from left


to right using process of elimination until the first group
consistent with the laboratory data is identified.

3. SHORT CUT: If the percent passing the #200 sieve is less than
or equal to 35%, use the AASHTO Classification Table starting
at column A-1-a. (Short cut: ONLY if A-1 and A-3 are
eliminated—Figure 2.3 using the LL and PI).

4. SHORT CUT: If the percent passing the #200 sieve is greater


than 35%, use the AASHTO Classification Table starting at
column A-4. (Short cut: Figure 2.3 using the LL and PI).

5. Calculate the group index or partial group index using the given
equations (below Table 2.1). If the equation yields a negative
number, report it as zero. Always round the Group Index (GI) to
the nearest whole number.

6. SHORT CUT: Groups A-1, A-3, A-2-4 and A-2-5: GI = 0.


Groups A-2-6 and A-2-7: use PGI equation.

7. Reference CERM (2018) Chapter 35.3, Das (2007) Ch 1.7,


and/or McCarthy (2007) Ch 4.4.

12
Refresher Class Geotechnical

Table 2.1: AASHTO Classification Table (CERM Table 35.4)

IMPORTANT TEST
TIP:

Do not use this chart


unless A-1-a, A-1-b, and
A-3 columns have been
eliminated. For example,
when the F200 < 35%
and PI < 6, Table 2.1
should be used instead of
this Figure.

Figure 2.3: AASHTO Graphical Chart (See also McCarthy Figure 4-17b, p.105)
Coarse-Grained Soils (F200  35%): Use A-2-4, A-2-5, A-2-6, and A-2-7
Fine-Grained Soils (F200 > 35%): Use A-4, A-5, A-6, A-7-5, and A-7-6

13
Refresher Class Geotechnical

EXAMPLE 2.2
Determine the AASHTO Classification of soils A & B & C
Sieve No % Passing % Passing % Passing
Soil A Soil B Soil C
No. 10 82 74 100
No. 40 46 52 90
No. 200 23 30 87
LL NP 25 71
PL NP 14 40

Solution:

Sample A: IMPORTANT TEST TIP:

LL = NP PI = NP Pay attention to the difference


between the “PL” and “PI”. If
A) A-1-a the plastic limit is given:
B) A-1-b PI = LL - PL
C) A-3
D) A-2-4

GI = ______

AASHTO Classification: _____

Sample B: Sample C:

LL = 25 PI = _____ LL = 71 PI = _____

A) A-2-4 A) A-4
B) A-2-5 B) A-5
C) A-2-6 C) A-6
D) A-2-7 D) A-7  A-7-5 or A-7-6?

GI = ______ GI = ______

Classification: ______ Classification: _____

(Answers and Extended Solution given in Appendix A)

14
Refresher Class Geotechnical

2.4 Unified Soil Classification System (USCS)

 Particle Size Definitions

Gravel: 75 mm (3 in.) to 4.75 mm (No. 4)


Sand: 4.75 mm (No. 4) to 0.075 mm (No. 200)
Silt and Clay: Less than 0.075 mm (No. 200)

 Group Symbols

First Letter: G Gravel


S Sand
M Silt
C Clay
O Organic

Second Letter: For Granular Soils - “G” or “S”


P Poorly Graded
W Well Graded
M Silty
C Clayey

For Fine-Grained Soils – “M”, “C” or “O”


L Low Plasticity
H High Plasticity or Elastic

Other: PT Peat

15
Refresher Class Geotechnical

 Classification Procedures:

1. Identify the percent gravel, percent sand and percent fines (using
No. 4 and No. 200 sieves).

2. If the percent passing the No. 200 sieve is greater than or equal
to 50%, then the sample is “fine-grained”.

 Determine the LL and PI and plot results on Casagrande


Plasticity Chart (Fig 2.6). Use Fig 2.5 to determine group
symbol. Note that “non-plastic” soil (PI < 4) classifies as silt
(usually ML).

3. If the percent passing the No. 200 sieve is less than 50%, then
the soil is “coarse-grained”.

 For soils with less than 5% fines, determine Cu & Cz and


use Fig 2.4 to determine group symbol.

 For soils with greater than 12% fines, determine the LL and
PI of fraction passing the No. 40 sieve and plot results on
the Casegrande Plasticity Chart (Fig 2.6). Use Fig 2.4 to
determine group symbol. Note that if the fines plot in the
“CL-ML” area, the group symbol will either be GC-GM or
SC-SM.

4. If the soil has 5 to 12 % fines, the soil will have a dual symbol.

 Determine the Cu & Cz to determine the first symbol


(GP/GW or SP/SW). Determine the LL and PI of fraction
passing the No. 40 sieve and plot the results on the
Casagrande Plasticity Chart (Fig. 2.6) to determine the
second symbol (GM/GC or SM/SC). Use Fig 2.4 to
determine group symbol.

 Reference CERM (2018) Ch 35.4, Das (2007) Ch 1.7, and/or


McCarthy (2007) Ch 4.4.

16
Refresher Class Geotechnical

Figure 2.4: USCS Coarse-Grained Classification Flow Chart


(% Passing No. 200 Sieve < 50%)
(adapted from ASTM International. See also Das (2007), McCarthy (2007), or Coduto (1999))

17
Refresher Class Geotechnical

Figure 2.5: USCS Inorganic Fine-Grained Classification Flow Chart


(% Passing No. 200 Sieve  50%)
(adapted from ASTM International. See also Das (2007), McCarthy (2007), or Coduto (1999))

70

60

50
Plasticity Index (PI)

40
CH
30

MH
20
CL
CL-ML
10

ML
0
0 10 20 30 40 50 60 70 80 90 100
Liquid Limit (LL)

Figure 2.6: Casagrande’s Plasticity Chart


(adapted from ASTM International. See also Das (2007), McCarthy (2007), or Coduto (1999))

18
Refresher Class Geotechnical

EXAMPLE 2.3
Determine the USCS Classification of soils A & B & C

Sieve No % Passing % Passing % Passing


Soil A Soil B Soil C
No. 4 100 98 77
No. 40 90 46 42
No. 200 87 23 10
LL 75 20 22
PI 20 6 5
Cu 26
Cc 0.2

Solution:
Sample A:
LL = 75 PI = 20

Gravel = _____ % Sand = _____ % Fines = _____ %

Coarse-grained or fine-grained?

Using flow chart for fine-grained soils (Fig 2.5), as needed, or just use
Casegrande Chart (Fig 2.6):

1. LL > 50
2. Plots below A-Line on Casagrande Chart (Fig 2.6)

USCS Group Symbol = _____

Sample B:
LL = 20 PI = 6

Gravel = _____ % Sand = _____ % Fines = _____ %

Coarse-grained or fine-grained?

19
Refresher Class Geotechnical

Using flow chart for coarse-grained soils (Fig 2.4):

1. % sand > % gravel


2. > 12 % fines
3. Refer to Casagrande Chart (Fig 2.6): fines = CL-ML

USCS Group Symbol = _____

Sample C:
LL = 22 PI = 5

Gravel = _____ % Sand = _____ % Fines = _____ %

Coarse-grained or fine-grained?

Using flow chart for coarse-grained soils (Fig 2.4):

1. % sand > % gravel


2. 5% < % fines<12%
3. Cu > 6 and Cz < 1 (therefore poorly-graded)
4. Refer to Casagrande Chart (Fig 2.6): fines = CL-ML

USCS Group Symbol = _____

(Answers given in Appendix A)

20
Refresher Class Geotechnical

2.5 USDA Classification System

 Particle Size Definitions

Coarse Fragments: Greater than 2.0 mm (No. 10)


Sand: 2.0 mm (No. 10) to 0.05 mm
Silt: 0.05 mm to 0.002 mm
Clay: Less than 0.002 mm

 Procedure

1. Determine the percent sand, silt, and clay (if the percent
passing No. 10 is less than 100 percent, the sample must be
normalized).

2. Plot results on USDA Classification Chart (Fig 2.7).

 Reference McCarthy (2007) Ch 4.4 and/or Coduto (1999) Ch 5.1.

Figure 2.7: USDA Soil Texture Classification Chart


(Source: United States Department of Agriculture)

21
Refresher Class Geotechnical

EXAMPLE 2.4
The results of a sieve analysis for two soil samples are shown below.
Determine the USDA classification for both samples.

Sieve Size Percent Percent


Finer Finer
Sample A Sample B
No. 10 100 85
No. 200 90 76
0.05 mm 85 68
0.002 mm 60 21

Solution:

Sample A:

Sand = _____%
Silt = _____%
Clay = _____%

USDA Classification: ___________

Sample B:

Coarse Fraction = _____%


Sand = ____% Normalized = _____ %
Silt = ____% Normalized = _____ %
Clay = ____% Normalized = _____ %

USDA Classification: ___________

(Answers given in Appendix A)

22
Refresher Class Geotechnical

3.0 Soil Sampling and In-Situ Testing Methods

3.1 Undisturbed Sampling

 The objective of undisturbed sampling is to keep the in-place


structure of the soil intact during sampling and transportation.

 Undisturbed samples are required for in-situ unit weight,


permeability, compressive strength, shear strength and consolidation
tests (see Table 3.4). They are generally only collected for softer
cohesive soils.

 The most common method used in the U.S. to obtain undisturbed


samples is to push a thin-walled (Shelby) tube sampler into the
undisturbed soil, and extract the tube with the sample inside. The
diameter and wall thickness can vary, but generally should have an
helby Tube area ratio less than 10%. The area ratio is a function of the sampler
outer diameter (o.d.) and inner diameter (i.d.), and defined as:

(o.d .) 2  (i.d .) 2
Area ratio (%)   100%
(i.d .) 2

 Other undisturbed sampling methods include thin-walled piston


samplers and block sampling.

3.2 Disturbed Sampling

 Disturbed sampling does not require that the in-place structure of the
soil remain in-place.

 Split-barrel (split-spoon) samplers are commonly used to obtain


disturbed samples.

23
Refresher Class Geotechnical

Figure 3.0: Hollow-Stem Sampling

3.3 Rock Quality

 Rock Quality Designation (RQD)

Σ lengths of intact pieces of core >100 mm


RQD =  100%
length of core advance

 Correlation between RQD and quality description:

Table 3.0: Descriptive Terms of Rock Quality

RQD (%) Rock Quality


0 to 25 Very Poor
25 to 50 Poor
50 to 75 Fair
75 to 90 Good
90 to 100 Excellent

24
Refresher Class Geotechnical

3.4 Common In-Situ Tests

Table 3.1: Summary of Common In-Situ Tests for Soils


(after FHWA)
Test Suitable for Not suitable for Properties that can be
estimated
SPT All soils finer Gravel Stratigraphy, undrained
than gravel: may shear strength in clay,
require retainer relative density and friction
in granular soils angle in sand

CPT Sand, silt, clay, Gravel; soils Continuous evaluation of


and peat with gravel or stratigraphy, strength of
cobbles sand, undrained shear
strength of clay, relative
density; with piezocone also
estimate in situ stress, pore
pressure
FVT Soft to medium Sand and Undrained shear strength
clay, some silt gravel
and peat

PMT Clays, silt; soft Soft, sensitive Strength, Ko, OCR, in situ
rock, dense clays, loose stress, lateral
sand, non silts and sands compressibility, elastic
sensitive clay, modulus (E) and shear
gravel, and till modulus (G)
DMT Sand, silt, clay, Gravel Soil type, stratigraphy, Ko,
and peat OCR; undrained shear
strength, compressibility at
small strain, and elastic
modulus (E) in clay, friction
angle in sand
SPT = Standard penetration test (ASTM D1586)
CPT = Cone penetration test (ASTM D3441)
FVT = Field vane test (ASTM D2573)
PMT = Pressuremeter test (ASTM D4719)
DMT = Flat plate dilatometer test

25
Refresher Class Geotechnical

3.5 Standard Penetration Test (SPT)

Standard Penetration Test (SPT) Procedures

The standard penetration test (SPT) is widely used in the United States. This
test uses a split-spoon sampler whose inner diameter is commonly 1.375
inches.

The spoon is driven into the ground with a drop hammer. The hammer
weighs 140 pounds and the drop is 30 inches. The test involves driving the
hammer three consecutive increments of 6 inches into the undisturbed soil.
The field N-value is taken as the sum of the blows for the 2nd increment (6 to
12 inches) and 3rd increment (12 to 18 inches), representing the number of
blows per foot.

The measured field N-value is generally inconsistent due to equipment and


operator variability. Therefore an energy correction is applied to the
measured field N-value (Nm) to achieve a standardized N-value (N60).

E
N60  Nm
60

Additionally, because the SPT is sensitive to overburden pressure, some


applications require that the N-value also be corrected for the influence of
overburden pressure (generally only in cohesionless soils).

pa
N corr  N 60
 'v

where: Nm = field measured SPT N-value (blows per foot)


N60 = N-value corrected for driving energy
Ncorr = N-value corrected for overburden pressure
E = the driving energy (hammer efficiency) measured in
the field
pa = atmospheric pressure (14.7 psi  2,000 psf)
 ’v = effective overburden pressure

26
Refresher Class Geotechnical

EXAMPLE 3.0 IMPORTANT TEST TIP:

If a SPT test yields the following data, A standard split-spoon


what is the field N-value? sampler is often 24 inches in
length, therefore a 4th sample
1st increment (0 to 6 in.): 3 blows is sometimes taken for
information.
2nd increment (6 to 12 in.): 7 blows
3rd increment (12 to 18 in.): 6 blows The SPT N-value is
4th increment* (18 to 24in.): 9 blows ALWAYS the sum of the 2nd
and 3rd increment.

Solution:

N-value is the sum of the blows for the 2nd & 3rd increments:

Nm = 7 + 6 = 13

Table 3.2: Strength of Cohesive Soils (Source: Coduto, 1999)


Unconfined
Compressive SPT, N
Descriptive Term Strength, tsf bpf
Very soft < 0.25 0 to 2
Soft 0.25 to 0.50 2 to 4
Medium stiff 0.50 to 1.00 4 to 8
Stiff 1.00 to 2.00 8 to 15
Very stiff 2.00 to 4.00 15 to 30
Hard > 4.00 > 30

Table 3.3: Relative Density of Granular Soils (CERM Table 35.9)


SPT N- Relative Internal Friction Angle,
Value Density 
Type of Soil (bpf) (%) Peck Meyerhof
Very loose sand <4 < 20 < 29 < 30
Loose sand 4 – 10 20 – 40 29 – 30 30 – 35
Medium dense sand 10 – 30 40 – 60 30 – 36 35 – 40
Dense sand 30 – 50 60 – 80 36 – 41 40 – 45
Very dense sand > 50 > 80 > 41 > 45

27
Refresher Class Geotechnical

Figure 3.1: Sample boring log

(Note that Qp is an estimation of the unconfined compressive strength, qu,


determined by hand penetrometer.)

28
Refresher Class Geotechnical

EXAMPLE 3.1
Use the boring log in Figure 3.1, shown on the previous page to answer
the following:

A) Which descriptive term would be most appropriate for the fat clay layer
encountered between the depths of 3.5 and 8.5 feet:

a) very loose
b) very soft
c) loose
d) soft

B) The best estimate of the shear strength parameter  of the sand layer
encountered at EL 776.5 to EL 771.5:

a) 28
b) 35
c) 42
d) 49

C) The estimated unconfined compressive strength (tsf) of the lean clay


encountered at a depth of 1.0 to 3.5 feet is most nearly:

a) 0.5
b) 0.75
c) 1.0
d) 1.25

D) Which rock quality description term best describes the limestone


encountered at a depth of 35 feet:

a) Poor
b) Fair
c) Good
d) Excellent

(Answers given at the end of Refresher Notes)

Note: See Chapter 9.0 of Refresher Notes for discussion on foundation types

29
Refresher Class Geotechnical

3.6 Common Laboratory Tests

Table 3.4: Summary of Common Laboratory Tests for Soils


Appropriate
Test Disturbed Undisturbed Properties Measured
for
Water content, indicator
Water
of consistency, plasticity,
Content, X silt and clay
shear strength, and
D2216
compressibility

Unit Weight, cohesive silt Dry unit weight and water


X
D2166 and clay content

Liquid Limit, Transition from plastic to


X plastic soils
D4318 liquid state

Plastic Limit, Transition from semi-


X plastic soils
D4318 solid to plastic state

Shrinkage Transition from solid to


X plastic soils
Limit, D427 semi-solid state

Specific
X all soils Specific gravity, Gs
Gravity, D854

Falling Head Hydraulic conductivity, k,


fine sand, fine
Permeability, X soils with
grained soils
D5084 k < 10-3 cm/sec
Constant
Hydraulic conductivity, k,
Head coarse
X soils with
Permeability, grained soils
k > 10-3 cm/sec
D5084

Consolidation, Preconsolidation stress,


X clay
D2435 settlement properties

30
Refresher Class Geotechnical

Table 3.4: Summary of Common Laboratory Tests for Soils (Continued)


Appropriate
Test Disturbed Undisturbed Properties Measured
for

Unconfined compressive
Unconfined medium stiff,
strength, qu, undrained
Compression, X stiff, and some
shear strength
D2166 very stiff clays
(cohesion), su = qu/2

With pore pressure


CU Triaxial
measurements, total, c, ,
Compression, X sand, silt, clay
and effective, c’, ’, shear
D4767
strengths
CD Triaxial Effective shear strength,
Compression, X sand, silt, clay c’, ’ (pore pressure is not
D7181 measured)

Direct Shear, Shear strength (drainage


X sand, silt, clay
D3080 is not controlled), c, 

Proctor, Std,
Moisture-Density
D698, Mod, X sand, silt, clay
Relationship
D1557

Relative
coarse sand,
Density, X Relative density, Dr
gravel
D4253

Mechanical Fraction of particle size


X gravel, sand
Sieve, D422 distribution > 0.075 mm

fine grained
Hydrometer, Fraction of particle size
X soils
D422 distribution < 0.075 mm
(silt, clay)

31
Refresher Class Geotechnical

4.0 Compaction and Pavement Design Parameters

 Compaction is densification of soil by the reduction of air in the soil


voids. The degree of compaction is measured in dry unit weight (dry
density).

 Purposes of compaction:
 Reduces subsequent settlement under working loads.
 Increases the shear strength of the soil.
 Reduces the voids ratio making it more difficult for water to flow
through soil, (ex: earth dams or pond line).
 Prevents the buildup of large pore water pressures that cause
soil to liquefy during earthquakes.

 Factors affecting compaction:


 Water content of the soil.
 The type of soil being compacted.
 The amount of compactive energy (lift thickness, number of roller
passes, weight of roller).
 Type of compaction equipment (smooth drum, sheepsfoot,
pneumatic tire, vibrating or static, etc.).
 Speed of application.

Figure 4.0: Moisture-unit weight relations varied by soil type (left), and
varied by compaction energy (right).
(Note: These curves are conceptual and not based on real data.)

32
Refresher Class Geotechnical

Figure 4.1: Application of compaction equipment by soil type.


(Source: Coduto, 1999, p. 173)

4.1 Proctor Laboratory Tests

 Standard Proctor Test (ASTM D698) and Modified Proctor Test


(ASTM D1557)

 Theoretical Energy

(Wt )(Drop)(Blows)(Layers)
Theoretical Energy 
Volume

(5.5 lbs)(1 ft)(25)(3)


Std. Proctor:   12,375 lb
1 / 30 ft 3 ft 2

(10 lbs)(1.5 ft)(25)(5)


Mod. Proctor:   56,250 lb
1/ 30 ft 3 ft 2

 Proctor curve cannot plot above the “zero voids” line, which is a plot
of dry unit weight (d) vs. moisture content (w), at 100 percent
saturation (S=100%). At 100 percent saturation the volume of air is
zero (Va=0), therefore there can be no further reduction in air voids.

 Reference CERM (2018) Chapters 35.11 and 35.12.

33
Refresher Class Geotechnical

Figure 4.2: A typical compaction test proctor curve

EXAMPLE 4.0
Standard proctor test results are given in the following:

Volume of Mold Wt. of Moist Soil Moisture Content


(ft3) (lb) (%)
1/30 3.53 11
1/30 3.85 13
1/30 4.01 15
1/30 3.97 17
1/30 3.77 19
1/30 3.69 21

Determine the maximum dry unit weight of compaction and the optimum
moisture content.

34
Refresher Class Geotechnical

Solution:
Prepare the following table:

Volume of Wt. of Moist Unit Moisture Dry Unit


Mold Moist Soil Weight Content Weight
(ft3) (lb) (lb/ft3) (%) (lb/ft3)
1/30 3.53 11
1/30 3.85 13
1/30 4.01 15
1/30 3.97 17
1/30 3.77 19
1/30 3.69 21

Use the data in this table to estimate the dry unit weight (d) vs. moisture
content (w).

Maximum dry unit weight = _____ pcf

Optimum Moisture Content = _____%

The proctor curve for these data is shown below:

106

104

102
Dry Unit Weight, pcf

100

98

96

94

92

90
10 12 14 16 18 20 22
Water Content, %

(Answers given in Appendix A)

35
Refresher Class Geotechnical

4.2 Field Density Tests

 Rubber Balloon Method (ASTM D 2167) & Sand Cone Method (ASTM D 1556)

 Often used with Speedy Moisture Test (ASTM D 4944)

Figure 4.3: Rubber Ballon Test Method Sand Cone Test Method
(https://labmekanikatanah.wordpress.com/2013/10/29/field-test-rubber-balloon-method/) (http://www.concrete-catalog.com/soil_compaction.html)

 Rubber Balloon Test Method: Vhole is known from calibrated Vessel

 Sand Cone Test Method: Known Information:


 Unit weight of dry uniform sand in jar (sand).
 Volume of cone (Vcone) or weight of sand to fill cone (Wcone).

Measured Information:
 Weight of jar with sand before test (Wo).
 Weight of jar with sand after test (Wf).
 Total weight of soil excavated from hole (Whole).
 Moisture content of the soil excavated from the hole (w).

Determine Dry Unit Weight of Excavated Soil:


 Volume of soil excavated from the hole is solved for by:

(Wo  W f )  Wcone (Wo  W f )


Vhole    Vcone
 sand  sand

 Dry unit wt. of soil excavated from the hole solved for by:
 W hole
d  
1 w V hole (1  w )

36
Refresher Class Geotechnical

 Nuclear density testing (ASTM D 6938)


 Nuclear density testing is a rapid, non-destructive testing
method for in-place measurements of wet and dry density and
water content of soil/soil-aggregate via direct or backscatter
methods (see Figure 4.4).

 Used for quality control and acceptance testing of compacted


soil and soil-aggregate mixtures (i.e. bituminous asphalt). This
method is inappropriate for testing virgin sub-grade.

 Assumes the material is homogeneous, therefore


anomalies, such as irregular aggregate size, irregular voids, or
deleterious materials (i.e. construction debris, heavy metals,
organic soils, etc.) can result in false readings.

 Density: assumes the gamma ray (photon) will lose energy and
rebound in a different direction as it interacts with the orbital
electron. This is called “Compton scattering”.

 Water content: assumes hydrogen ions present in the soil/soil-


aggregate is in the form of water (H2O).

 Effect of frost: Moisture content will be exaggerated and dry


density reading will be lower.

 Effect of lime-treatment: Lime treatment adds many hydroxyl


ions to soil. If the nuclear density gauge is not properly
recalibrated, it will interpret these as hydrogen atoms (i.e.
water), which will result in falsely high water content reading
and a falsely low dry density reading.

Figure 4.4: Typical Schematic of a Nuclear Density Gauge


(Source: http://www.concrete-catalog.com/soil_compaction.html)

37
Refresher Class Geotechnical

4.3 Relative Compaction

 Relative compaction is the ratio of the field dry unit weight to the
maximum dry unit weight.

Field Dry Unit Weight ( d ) field


RC    100%
Max Dry Unit Weight ( d ) max

EXAMPLE 4.1
A field sand cone test is performed on a layer of compacted fill. The results
of the sand cone test are shown below. A laboratory Proctor test
determined that the fill soil has a maximum dry unit weight of
approximately 120 pcf. Determine the relative compaction of the fill.

Unit weight of dry uniform of sand, sand: 95 pcf


Wt of sand to fill cone, Wcone: 1.10 lb
Wt of jar + cone + sand (before test), Wo: 16.5 lb
Wt of jar + cone + sand (after test), Wf: 10.5 lb
Wt of moist soil excavated from hole, Whole: 6.60 lb
Moisture content of soil from hole, w: 10 %

Solution:
Calculate volume of soil excavated from the hole:

(Wo  W f )  Wcone 16.5 lb  10.5 lb  1.10 lb


Vhole    0.0516 ft 3
 sand 95 lb 3
ft

Calculate dry unit weight of soil excavated from the hole:

Whole 6.60 lb
d    116.3 lb
Vhole (1  w) (0.0516 ft 3 )(1  0.10) ft 3

Calculate the relative compaction:

( d ) field 116.3 pcf


RC   100%   100%  97%
( d ) max 120 pcf

38
Refresher Class Geotechnical

4.4 California Bearing Ratio

 CBR test is used only for flexible pavement design (bituminous).

 (ASTM D1883) is an empirical measure of mechanical subgrade


strength compared to that of high quality crushed rock (CBR = 100).
The harder the surface, the higher the CBR rating.

 The test is performed by measuring the pressure required to


penetrate a soil sample with a plunger of standard area (3 sq. in.). In
the laboratory, the sample is compacted into the CBR mold and is
usually immersed in a water bath prior to testing. Load is applied to
the plunger and the applied stress and penetration for a series of
loads are recorded and plotted.

 The CBR rating is calculated for the stress at 0.1 and 0.2 inches of
penetration by the formulas below, where 1000 psi and 1500 psi are
the standard stresses for 0.1 and 0.2 inches of penetration,
respectively:

actual stress
CBR  100
standard stress

 0.1  0.2
CBR0.1   100 CBR0.2   100
1000 psi 1500 psi

 The CBR rating is typically taken as the CBR0.1 (in percent and
rounded to the nearest whole number), where the CBR 0.2 is less
than the CBR0.1.

 Note: If CBR0.2 is greater than CBR0.1 the sample must be retested.


If the retest results are consistent, then the CBR rating is taken as
the CBR0.2 (in percent and rounded to the nearest whole number).

 Reference CERM (2018) Chapter 35.22.

39
Refresher Class Geotechnical

4.5 Subgrade Modulus

 Rigid slabs are designed based on the subgrade modulus


(modulus of subgrade reaction), k.

 The subgrade modulus is determined using the plate bearing


load test, which is performed in-situ on compacted soil using a
standard 30-inch diameter steel plate. The plate is loaded a
series of 10 times.

 The corrected load verses the corrected deflection is graphed for


the 10th repetition. The subgrade modulus is taken as the slope
of the line of stress vs. deflection.

 Figure 4.5 can be used to correlate the standard modulus of


subgrade reaction (Line C) to the CBR (Line A) and typical values
for USCS soil types (Line D)

 Reference CERM (2018) Chapter 35.23.

Figure 4.5: Correlations of Subgrade Modulus to CBR


(Source: ACI 360R-92, 1997 p. 12)

40
Refresher Class Geotechnical

4.6 Resilient Modulus

 The Resilient Modulus (MR) is a subgrade material stiffness test that


is primarily used for the design of flexible pavements.

 A material's resilient modulus is actually an estimate of its modulus


 of elasticity (E), where the resilient modulus is stress divided by strain
for rapidly applied loads (like those experienced by pavements).

 The MR can be correlated using the widely used empirical relationship


with the CBR value (applicable only for fine-grained soils with CBR 
10):

M R ( psi )  (1500)CBR
(CERM 2018, pp. 76-20)

 The AASHTO Design Guide uses the following correlation:

M R ( psi)  (2555)CBR0.64
(CERM 2018, pp. 76-22)

 Reference CERM (2018) Chapter 35.22

Table 4.0: Typical Resilient Modulus Values

Typical Range of
Material Resilient Modulus, MR (psi)
Crushed Stone 20,000-40,000
Silty Soils 5,000-20,000
Clay Soils 5,000-15,000

41
Refresher Class Geotechnical

4.7 Expansive Soils

 Expansive proterty of soil is related to swell potential which is a


function of clay minerals present in a given soil. Swelling occurs when
water particles infiltrate in and between clay minerals.

 Swell Potential:

Montmorillonite > Illite > Kaolinite

 Foundations bearing on expansive soil will be subjected to heave –


uplift of the foundation – and differential settlement problems.

 Ground improvement tecniques to decreae heave:


Chemical soil stabilization (Use of lime and cement)
Installation of moisture barrier (Prevention of water flow)
Compaction of expansive soil (Lower permeability)
Removal of expansive soil under foundation

4.7 Frost Susceptibility

 Frost susceptibility is generally a function of the soil type.

 Soils that are highly susceptible must be capable of drawing


significant amounts of water up into the “frost zone” through capillary
action.

 Silts are generally the most susceptible to frost heave. (Note:


some clays may have small D10, but permeability is too low to draw
water upward)

 The height of capillary rise in sands and silts can generally be taken
as (Holtz & Kovatz, 1981):

0.15
hc ( m) 
D10 (mm)

where: hc = Height of capillary rise (m)


D10 = Effective grain size (mm)

42
Refresher Class Geotechnical

5.0 Permeability & Seepage

5.1 Darcy’s Law and Bernoulli’s Equation

 Darcy’s law is defined as the following:

Q  vA  kiA

where: Q = Flow rate


v = Average discharge velocity (v = ki)
A = Cross-Sectional area
k = Coefficient of permeability
i = Hydraulic gradient

 The hydraulic gradient is the head loss between two points.

h
i
l

where: i = Hydraulic gradient


h = Loss in total head
l = Distance water travels

Figure 5.0: Hydraulic Gradient as Head Loss between Two Points


(Source: adapted from Coduto, 1999)

43
Refresher Class Geotechnical

 In soil and rock applications, total head is defined as the following:

h  hz  h p

where: h = Total head


hz = Elevation head, which is the difference between the
datum and a given point (potential energy).
hp = Pressure head, which is the difference between a
given point and the water level shown in the
piezometer (strain energy).

44
Refresher Class Geotechnical

Figure 5.1: No Flow

 Hydraulic gradient in the sand column with no flow:

h 0 ft
i  0
l 2 ft

Note that there is no change in total head, therefore when there is


no flow the hydraulic gradient is always equal to zero.

 Calculating head in the sand column with no flow:

Total head at the top of the sand column:

h  h z  h p  4 ft  1 ft  5 ft

Total head at the bottom of the sand column:

h  h z  h p  2 ft  3 ft  5 ft

45
Refresher Class Geotechnical

Figure 5.2: Downward Flow

 Hydraulic gradient in the sand column with downward flow in the


direction flow (top to bottom):

h 5 ft  4 ft
i   0.5
l 2 ft
Note that the only change in head occurs within the sand column.

 Calculating head in the sand column with downward flow:

Total head at the ENTRANCE (top) of the sand column:

h EN T  h z  h p  4 ft  1 ft  5 ft

Total head at the EXIT (bottom) of the sand column:

h EXT  h z  h p  2 ft  2 ft  4 ft

Total head and pressure head at the center of the sand column:

hMID  hENT  i  l  5 ft  0.5(1 ft)  4.5 ft


h p  hMID  hz  4.5 ft  3 ft  1.5 ft

46
Refresher Class Geotechnical

Figure 5.3: Upward Flow

 Hydraulic gradient in the sand column with upward flow in the


direction flow (bottom to top):

h 6 ft  5 ft
i   0.5
l 2 ft

 Calculating head in the sand column with upward flow:

Total head at the ENTRANCE (bottom) of the sand column:

h EN T  h z  h p  2 ft  4 ft  6 ft

Total head at the EXIT (top) of the sand column:

h EXT  h z  h p  4 ft  1 ft  5 ft

Total head and presseure head at the center of the sand column:

hMID  hENT  i  l  6 ft  0.5(1 ft)  5.5 ft


h p  hMID  hz  5.5 ft  3 ft  2.5 ft

47
Refresher Class Geotechnical

EXAMPLE 5.0
Determine the elevation, pressure and total heads at the top and bottom
of the sand column, and at Point A of the figure below.

Solution:
Total head at the ENTRANCE of the sand column:
h E N T  h z  h p  ____ ft  ____ ft  ____ ft

Total head at the EXIT of the sand column:


hEXT  hz  h p  ____ ft  ____ ft  ____ ft

Gradient through sand column in the direction of flow:

h 6 ft  3 ft
i   1.0
l 3 ft

Total head and pressure head at Point A:


h A  hENT  i  l  ___ ft  (____ )( ___ ft)  ___ ft

( h p ) A  h A  ( hz ) A  ____ ft  ____ ft  ____ ft

(Answers given in Appendix A, alternative solution given in Appendix B)

48
Refresher Class Geotechnical

5.2 Seepage Velocity

 Seepage velocity is the rate of movement of an element of water


through a soil.
ki v
vs  
ne ne

where: vs = Seepage velocity


v = Average discharge velocity
k = Hydraulic conductivity (coeff. of permeability)
i = Hydraulic gradient
ne = Effective porosity (for sandy soils ne  n)

 The effective porosity considers only interconnected pore spaces (for


sandy soils ne  n).

5.3 Coefficient of Permeability Laboratory Tests

 Constant head permeability test:

∆ℎ
Soil Sample

Water colleted V
Volume = (V)

Figure 5.4: Constant Head Apparatus

49
Refresher Class Geotechnical

 Appropriate for coarse-grained soils with permeability greater than


10-3 cm/sec. The coeff. of permeability via constant head is solved for
by:

VL
k
hAt

where: V = Volume of water


L = Length of specimen between piezometers
A = Cross-sectional area of specimen
t = Duration of water collection
h = Head difference

EXAMPLE 5.1
A constant-head permeability test was performed on a 110 mm diameter,
270 mm tall fine sand sample in a permeameter similar to the one shown
in Figure 5.4. The piezometers are spaced 200 mm apart and had
readings of 1809 and 1578 mm. The graduated cylinder collected 910 ml
of water over 25 min 15 sec. Calculate the hydraulic conductivity of the
soil in cm/sec.

Solution:

Define the following parameters:

V  910 ml  910 cm3


A  4 (11 cm) 2  95 cm 2
L  20 cm
h  180.9 cm  157.8 cm  23.1 cm
t  1515 sec

Solve for k:

VL (910 cm3 )(20 cm)


k  2
 5.5  103 cm
sec
hAt (23.1 cm)(95 cm )(1515 s)

50
Refresher Class Geotechnical

 Falling head permeability test:

a= X-sectional area of Standpipe

Time = to

Time = t1

h0
h1

Soil Sample
L

Volume of water - NOT required

Figure 5.5: Falling Head Apparatus

 Appropriate for fine-grained soils with permeability less than 10-3


cm/sec. The coeff. of permeability via falling head is solved for by:

 aL h   aL h0 
k   2.303 log10 0    ln 
 At h1   At h1 

where: h0 = Head at the start of the test (t0)


h1 = Head at the end of the test (t1)
L = Length of specimen
A = Cross-sectional area of specimen
a = Cross-sectional area of standpipe
t = (t1-t0) = Duration when water level falls from h0 to h1

51
Refresher Class Geotechnical

EXAMPLE 5.2
A falling-head permeability test was performed on a silty clay in a
permeameter similar to the one shown in Figure 5.5. The diameter and
length of the sample were 10.20 and 16.20 cm, respectively. The cross-
sectional area of the standpipe was 1.95 cm2. The water in the standpipe
dropped from a height of 100 cm at the start of the test to 92 cm at the
end of the test. The test duration was 35 minutes. Calculate the coefficient
of permeability in cm/sec.

Solution:
Define the following parameters:
A  81.7 cm 2
a  1.95 cm 2 h0  100 cm
L  16.2 cm h1  92 cm
t  2100 sec

Solve for coefficient of permeability:


aL h0
k ln
At h1
(1.95 cm 2 )(16.2 cm) 100 cm
k 2
ln  1.5  105 cm
sec
(81.7 cm )(2100 s) 92 cm

 Empirical relationships:

For natural, uniform sands and gravels, and non-plastic silty sands
within the range of 10-1 an7d 10-3 cm/s:
0.7825
 2 e 
3
k (cm / s )  2.4622  ( D10 , mm) 
 1  e 

where: D10 = Effective particle size (see CH2)


e = void ratio

 Reference CERM (2018) Chapter 35.15 and Das Chapter 1.10.

52
Refresher Class Geotechnical

5.4 Flow Nets

 Laplace’s Equation represents energy loss through a resistive


medium (i.e. flow through soil). A flow net is a 2D graphical solution
of Laplace’s Equation.

 A flow net is a combination of flow lines and equipotential lines.

 A flow line is a line along which a water particle travels.

 There is no flow along equipotential lines, which are 90 degrees to


flow lines. The total head along an equipotential line is equal at all
points.

 Rules of flow net construction:

 Flow lines cannot cross other flow lines and equipotential lines
cannot cross other equipotential lines.

 Equipotential lines intersect the flow lines at right angles.

 Flow net must be constructed so that each element is a


curvilinear square (sides may be curved but a circle must be
inscribed within it that touches all its 4 sides).

 Reference CERM (2018) Ch 21-15, Das (2007) Ch 1.9, and/or


McCarthy Ch 7.1 (2007).

 The total flow rate though a flow net, is solved for by:
Nf
Q  k h L
Nd

where: Q = Total flow rate


Nf = Number of flow channels in a flow net
Nd = Number of potential drops
h = Head change from upstream to downstream
k = Coefficient of permeability
L = Length of structure [i.e. bank-to-bank]

53
Refresher Class Geotechnical

EXAMPLE 5.3
For a flow net shown in the following figure, determine head loss at points
A, B, and C. The structure is approximately 100 feet long. Determine the
flow rate in ft3/min through the permeable layer. Assume hydraulic
conductivity, k = 1.6410-4 ft/min.

Solution:

Nf = ____ Nd = ____

Total head loss = ____

Head loss per drop = ____

Head loss at Pnt A = ____ Pnt B = ____ Pnt C = ____

Calculate seepage:

Nf ft 3
Q  k h L  __________
Nd min

(Answers given in Appendix A)

54
Refresher Class Geotechnical

6 Vertical Stresses and Pore Water Pressure

6.1 Total Vertical Stress

 Total vertical stress, v, is generated by gravitation force acting on


the soil mass.

 v    i zi

where: v = Total vertical stress


i = Total unit weight of the soil layer(s)
zi = Thickness of soil layer(s)

Figure 6.0: Example for Total Stress

 The total vertical stress at Point A for the soil profile shown in Figure
6.0 can be calculated as:

 v   1 z1   2 z 2

55
Refresher Class Geotechnical

6.2 Pore Water Pressure

 Pore water pressure is the result of buoyant force, u, exerted by water


in the soil mass.

 GENERAL CASE: Can be used in all scenarios, but must be used to


compute pore water pressure in seepage conditions (upward or
downward 1-D flow). Note that “piezometric head” or “piezometric
water elevation” measured at a given point is equal to the total head
at that point.

u   whp

where: u = Pore water pressure


w = Unit weight of water
hp = Pressure head at the point of interest (hp = h )

 HYDROSTATIC CASE: In hydrostatic conditions (no flow), the


pressure head is equal to the distance between the point of interest
and the groundwater surface (aka phreatic surface). The pore water
pressure is simplied as:

u   w zw

where: u = Pore water pressure


w = Unit weight of water
zw = Depth below the groundwater surface

Figure 6.1: Examples of pressure head for hydrostatic (left)


and seepage (right) cases

56
Refresher Class Geotechnical

EXAMPLE 6.0

Calculate the pore pressure at Point A for both cases shown below

(a) (b)

Solution:

(a) Hydrostatic condition (Left):

u   w z w  (_______)(_______)  __________psf

(b) Seepage Condition (Right):

u   whp  (_______)(_______)  __________psf

Note that u = 0 at Point B in both cases.

(Answers given in Appendix A)

6.3 Effective Vertical Stress

 Effective Stress is the portion of the total stress that will be supported
through grain contact. The vertical effective stress, ’v, in a soil
element at a depth, z, is the difference of the total vertical stress
imposed by the weight above, minus the pore water pressure, u.

57
Refresher Class Geotechnical

 Effective vertical stress, ’v:

 'v   v  u

where: v = Total vertical stress (lb/ft2 or kN/m2)


’v = Effective vertical stress (lb/ft2 or kN/m2)
u = Pore water pressure (lb/ft2 or kN/m2)

 Reference CERM (2018) Chapter 35.7 and Das (2007) Chapter 1.10.

EXAMPLE 6.1
Assuming hydrostatic conditions, determine the total and effective vertical
stresses at Points A and B for the soil profile shown:

Solution:

Total vertical stress:

Point A:  v  (62.4 lb
)(10 ft)  624 lb
ft 3 ft 2

Point B:  v  (624 lb
)  (130 lb3 )(20 ft)  3224 lb
ft 2 ft ft 2

Effective vertical stress:

Point A:  'v  624 lb


 (62.4 lb
)(10 ft)  0 lb
ft 2 ft 3 ft 2

Point B:  'v  3224 lb


 (62.4 lb
)(30 ft)  1352 lb
ft 2 ft 3 ft 2

58
Refresher Class Geotechnical

7 Pressure Distribution

7.1 Point Loads

 For pressure distribution analysis, we assume that soil is a semi-


infinite, homogeneous, linear, isotropic, elastic material.

 Boussinesq (1885) presented a solution for the distribution of


stresses for a point load applied on the surface.

 The increase in vertical stress () on a soil element located at depth


(z) and a radial distance (r) from the axis of the point load (P) are
given by:

2.5
 3P  1 

   
2  2
 2z  1  ( z ) 
r

 3P 
and when r = 0:     
 2 z 2 

 Reference CERM (2018) Chapter 40.1.

7.2 Uniformly Loaded Areas

 For uniformly loaded circular and rectangular areas, the increase in


vertical stress is determined by:

  qI  pI

where: q = Applied bearing pressure


I = Influence factor (see following Tables)

load
and: q
area

59
Refresher Class Geotechnical

UNIFORMLY LOADED CIRCULAR AREAS

r = radius of the uniformly loaded area


x = horizontal distance from the center of the uniformly loaded area
z = vertical depth below the base of the uniformly loaded area

Table 7.0: Influence factors for finding vertical stress under a circular
loaded area.
x/r
z/r
0.00 0.25 0.50 1.00 1.50 2.00 2.50 3.00 3.50 4.00
0.25 0.986 0.983 0.964 0.460 0.015 0.002 0.000 0.000 0.000 0.000

0.50 0.911 0.895 0.840 0.418 0.060 0.010 0.003 0.000 0.000 0.000

0.75 0.784 0.762 0.691 0.374 0.105 0.025 0.010 0.002 0.000 0.000

1.00 0.646 0.625 0.560 0.335 0.125 0.043 0.016 0.007 0.003 0.000

1.25 0.524 0.508 0.455 0.295 0.135 0.057 0.023 0.010 0.005 0.001

1.50 0.424 0.413 0.374 0.256 0.137 0.064 0.029 0.013 0.007 0.002

1.75 0.346 0.336 0.309 0.223 0.135 0.071 0.037 0.018 0.009 0.004

2.00 0.284 0.277 0.258 0.194 0.127 0.073 0.041 0.022 0.012 0.006

2.50 0.200 0.196 0.186 0.150 0.109 0.073 0.044 0.028 0.017 0.011

3.00 0.146 0.143 0.137 0.117 0.091 0.066 0.045 0.031 0.022 0.015

4.00 0.086 0.086 0.083 0.076 0.061 0.052 0.041 0.031 0.024 0.018

5.00 0.057 0.057 0.056 0.052 0.045 0.039 0.033 0.027 0.022 0.018

7.00 0.030 0.030 0.029 0.028 0.026 0.024 0.021 0.019 0.016 0.015

10.00 0.015 0.015 0.014 0.014 0.013 0.013 0.013 0.012 0.012 0.011

[Source: Liu and Evett, 2004]

60
Refresher Class Geotechnical

UNIFORMLY LOADED CIRCULAR AREAS

[Source: CERM, 2018 Appendix 40.B]

Figure 7.0: Boussinesq stress contours for uniformly loaded circular areas

61
Refresher Class Geotechnical

UNIFORMLY LOADED RECTANGULAR AREAS


m = B/z n = L/z
z = vertical depth below the base of the uniformly loaded area

Table 7.1: Influence factors for finding vertical stress under the corner of a
rectangular loaded area.
m or n
m or n 0.1 0.2 0.3 0.4 0.5 0.6 0.7 0.8 0.9 1.0
0.1 0.005 0.009 0.013 0.017 0.020 0.024 0.024 0.026 0.027 0.028

0.2 0.009 0.018 0.026 0.033 0.039 0.043 0.047 0.050 0.053 0.055

0.3 0.013 0.026 0.037 0.047 0.056 0.063 0.069 0.073 0.077 0.079

0.4 0.017 0.033 0.047 0.060 0.071 0.080 0.087 0.093 0.098 0.101

0.5 0.020 0.039 0.056 0.071 0.084 0.095 0.103 0.110 0.116 0.120

0.6 0.022 0.043 0.063 0.080 0.095 0.107 0.117 0.125 0.131 0.136

0.7 0.024 0.047 0.069 0.087 0.103 0.117 0.128 0.137 0.144 0.149

0.8 0.026 0.050 0.073 0.093 0.110 0.125 0.137 0.146 0.154 0.160

0.9 0.027 0.053 0.077 0.098 0.116 0.131 0.144 0.154 0.162 0.168

1.0 0.028 0.055 0.079 0.101 0.120 0.136 0.149 0.160 0.168 0.175

1.2 0.029 0.057 0.083 0.106 0.126 0.143 0.157 0.168 0.178 0.185

1.5 0.030 0.059 0.086 0.110 0.131 0.149 0.164 0.176 0.186 0.193

2.0 0.031 0.061 0.089 0.113 0.135 0.153 0.169 0.181 0.192 0.200

2.5 0.031 0.062 0.090 0.115 0.137 0.155 0.170 0.183 0.194 0.202

3.0 0.032 0.062 0.090 0.115 0.137 0.156 0.171 0.184 0.195 0.203

5.0 0.032 0.062 0.090 0.115 0.137 0.156 0.172 0.185 0.196 0.204

10.0 0.032 0.062 0.090 0.115 0.137 0.156 0.172 0.185 0.196 0.205

 0.032 0.062 0.090 0.115 0.137 0.156 0.172 0.185 0.196 0.205

[Source: Liu and Evett, 2004, p. 150]

62
Refresher Class Geotechnical

UNIFORMLY LOADED RECTANGULAR AREAS


m = B/z n = L/z
z = vertical depth below the base of the uniformly loaded area

Table 7.1 (Continued): Influence factors for finding vertical stress under
the corner of a rectangular loaded area.
m or n
m or n 1.2 1.5 2.0 2.5 3.0 5.0 10.0 
0.1 0.029 0.030 0.031 0.031 0.032 0.032 0.032 0.032
0.2 0.057 0.059 0.061 0.062 0.062 0.062 0.062 0.062
0.3 0.083 0.086 0.089 0.090 0.090 0.090 0.090 0.090
0.4 0.106 0.110 0.113 0.115 0.115 0.115 0.115 0.115
0.5 0.126 0.131 0.135 0.137 0.137 0.137 0.137 0.137
0.6 0.143 0.149 0.153 0.155 0.156 0.156 0.156 0.156
0.7 0.157 0.164 0.169 0.170 0.171 0.172 0.172 0.172
0.8 0.168 0.176 0.181 0.183 0.184 0.185 0.185 0.185
0.9 0.178 0.186 0.192 0.194 0.195 0.196 0.196 0.196
1.0 0.185 0.193 0.200 0.202 0.203 0.204 0.205 0.205
1.2 0.196 0.205 0.212 0.215 0.216 0.217 0.218 0.218
1.5 0.205 0.215 0.223 0.226 0.228 0.229 0.230 0.230
2.0 0.212 0.223 0.232 0.236 0.238 0.239 0.240 0.240
2.5 0.215 0.226 0.236 0.240 0.242 0.244 0.244 0.244
3.0 0.216 0.228 0.238 0.242 0.244 0.246 0.247 0.247
5.0 0.217 0.229 0.239 0.244 0.246 0.249 0.249 0.249
10.0 0.218 0.230 0.240 0.244 0.247 0.249 0.250 0.250
 0.218 0.230 0.240 0.244 0.247 0.249 0.250 0.250
[Source: Liu and Evett, 2004, p.155]

63
Refresher Class Geotechnical

Figure 7.1: Influence factors for finding vertical stress under the corner of
a rectangular loaded area. (Source: Liu & Evett, 2004, p. 156)

64
Refresher Class Geotechnical

UNIFORMLY LOADED RECTANGULAR AREAS

Figure 7.2: Influence factors for finding vertical stress under the corner of a
rectangular loaded area (Boussinesq Case) (Source: NAVFAC DM-7.01, 1986)

65
Refresher Class Geotechnical

UNIFORMLY LOADED RECTANGULAR AREAS – SPECIAL CASES

[Source: CERM, 2018 Appendix 40.A]

Figure 7.3: Boussinesq stress contours for uniformly loaded infinitely long
and square footings

66
Refresher Class Geotechnical

2:1 APPROXIMATE METHOD

 The 2V:1H approximate method of vertical stress distribution below


the foundation yields the average increase in vertical stress due to
an applied load across a plane below the bearing surface (not at a
discrete point)

 The average vertical stress increase at a depth z below a footing


(Point Load):
qBL P
Rectangular Area:   
( B  z )( L  z ) ( B  z )( L  z )
qr 2 P
Circular Area:   2
 2
 z
  r  
z
 r  
 2  2

Figure 7.4: 2:1 Approximate Method for Rectangular Area


(Reference CERM (2018) Chapter 40.2.)

 The average vertical stress increase at a depth Z below a continuous


footing (Line load):

𝑷(𝑳𝒊𝒏𝒆 𝒍𝒐𝒂𝒅 𝒖𝒏𝒊𝒕)


Infinetly Long foundation wall:
(𝑩 𝒁)(𝟏)

67
Refresher Class Geotechnical

EXAMPLE 7.0
A flexible rectangular area measures 10 by 20 ft in plan. It supports an
applied pressure of 3,000 psf. Determine the vertical stress increase due
to the applied load at a depth of 20 ft below the corner of the rectangular
area (Figure 7.1).

Solution:

B 10 ft L 20 ft
m   0.5 n   1.0  I
z 20 ft z 20 ft

  Iq  ( )(3000 psf )  360 psf

(Answers given in Appendix A)

EXAMPLE 7.1
A rectangular area measures 50 by 50 ft in plan. It supports a pressure
of 6,000 psf. Determine the increase in vertical stress due to the applied
load at a depth of 25 ft below the center of the rectangular area using the
Boussinesq stress contours (Figure 7.3).

Solution:

x = 0 ft
z = 25 ft
B = 50 ft

x 0 ft
Horizontal distance from center of footing:   x  0B
B 50 ft

z 25 ft
Vertical distance from base of footing:   z  0.5 B
B 50 ft

Using Fig 7.3, I = 0.7  = (0.7)(6000 psf) = 4200 psf

68
Refresher Class Geotechnical

OTHER APPROXIMATE METHODS


(BELOW CENTER OF LOADED AREA)

 The following approximations compute induced load below the center of


a loaded area. Results are typically within 5% of Boussinesq values. (q,
B, L, and z have been previously defined in this chapter; for circular
loads use diameter D = B)

 Circular loads:
  
1.50 
   
  1  
  q 1  
 2
   B  
  1   2 z   
   

 Square loads:
  
1.76 
   
  1  
  q 1  
 2
     B 
  1   2 z   
   

 Continuous loads:
  
2.60 
   
 1 
  q 1   

 1.38 
    B
 
  1   2 z  
   

 Rectangular loads:
  2.60 0.84 B / L 

   
 1 
  q 1   

 1.38 0.62 B / L 
   B  
  1    
 2z 
   

69
Refresher Class Geotechnical

8.0 Consolidation of Clay Soils

8.1 Overview of Settlement in Clay Soils

 Settlement of fine-grained soils occurs in three stages. Immediate


settlement occurs rapidly and is based on the theory of elasticity.
Primary consolidation occurs due the expulsion of water from soil
pores. Secondary compression (aka “creep”) occurs as soil particles
readjust and compress.

Immediate Settlement
Time
Primary Settlement Secondary/Creep Settlement

Immediate Settlement
Settlement

Primary Settlement

Secondary Settlement

Figure 8.0: Three phases of settlement in fine-grained soils

 Consolidation is the “primary consolidation” settlement phase of fine-


grained soils such as silts and clays due to low permeability. Primary
consolidation is due to dissipation of excess pore pressure (Δu) with
time. Coarse-grained soils (sands and gravels) undergo settlement
but at a much faster rate due to high permeability.

70
Refresher Class Geotechnical

 Seecondary compression occurs after all of the excess pore pressure


(Δu) dissipates and soil particles undergoes readjustment.

Compaction VS Consolidation

 The process of consolidation is often confused with the process of


compaction. Compaction increases the density of an unsaturated soil
by reducing the volume of air in the voids. Consolidation is a time-
related process of increasing the density of a saturated soil by
squeezing water from soil voids.

8.2 Stress History of Clay Soils

 Normally consolidated (NC) clay: the present effective overburden


pressure is the maximum pressure the soil has been subjected to in
the recent past.

 Overconsolidated (OC): the present effective overburden pressure is


less than what the soil has seen in the recent past. The past
maximum effective overburden pressure is called the
preconsolidation pressure.

1.6 A
1.5 B
(A B)
1.4 Recompression
(B C)
Curve
(Over-Consolidated) Virgin Curve
(Normally Consolidated)
Void Ratio (e)

1.3

1.2
1.1
D
1.0

0.9 (C D) C
Rebound Curve
(Over-Consolidated)
0.8

10 100 1000
Preconsolidation
Preessure (𝝈΄𝒄 )

Figure 8.1: Typical 1-D Consolidation Test

71
Refresher Class Geotechnical

 The stress history of soils is summarized by:

For normally consolidated soils:  ' 0   'c

For overconsolidated soils:  ' 0   ' c

where: ’o = Initial (present) effective overburden pressure


’c = Preconsolidation pressure

 'c
 The overconsolidation ratio (OCR) is defined as follows: OCR 
 'o

 Reference CERM (2018) Ch 40.7 and Das (2007) Chs 1.13 and 1.14.

8.3 Consolidation of NC Clays

B = Initial (Present) effective overburden Pressure

A C = Final effective overburden Pressure

B
Void Ratio (e)

∆𝑒
𝐶
D ∆𝑒
𝑐 =
𝜎
𝐿𝑜𝑔
C 𝜎
∆𝜎

𝝈𝒐 = (𝝈΄𝒄 ) 𝝈𝒇

𝜎 (𝐿𝑜𝑔 𝑆𝑐𝑎𝑒)

Figure 8.2: 1-D Consolidation Test of NC Clay

72
Refresher Class Geotechnical

 For a normally consolidated (NC) soil, primary consolidation


settlement can be calculated using the Compression Index, Cc:

 C   ' f 
S c    c  H log  

 1  eo    'o 

where: Sc = Primary consolidation settlement


 ’o = Initial effective overburden pressure
 ’f = Final effective overburden pressure (’o +)
H = Thickness soil layer
Cc = Compression index
eo = Initial void ratio

 In general, the compressible strata should be split into layers that are
a maximum of 15 feet thick for hand calculations and 5 feet thick for
computer calculations.

EXAMPLE 8.0
A square mat foundation will be
constructed at the ground surface of
the soil profile shown in the figure.
The foundation will increase the
vertical stress at the midpoint of the
normally consolidated clay layer by
450 psf. Determine the primary
consolidation settlement of the
normally consolidated layer.

73
Refresher Class Geotechnical

Solution:
Determine the initial and final effective overburden pressures at the
midpoint of the normally consolidated layer.

 '0  (120 lb ft3 )(10 ft)  (95 lb ft3  62.4 lb ft3 )(5 ft)  1363 psf

 ' f  1363 psf  450 psf  1813 psf

Determine the primary consolidation settlement of the normally


consolidated clay layer.

 Cc    ' f   0.2   1813 psf 


Sc    H log    (10 ft) log  
 1  eo    'o   1  0.65   1363 psf 

Sc  0.15 ft or 1.8 in

8.4 Consolidation of OC Clay

𝑪𝒓 = Initial Effective Stress


𝑪𝒓
= Final Effective Stress
∆𝑒
Void Ratio (e)

Void Ratio (e)

𝐶 =𝐶
𝑪𝒄

𝑪𝒔

Δe1 = Void ratio change in OC


Δe2 = Void ratio change in NC

𝜎 (𝐿𝑜𝑔 𝑆𝑐𝑎𝑒) 𝜎 (𝐿𝑜𝑔 𝑆𝑐𝑎𝑒)

Case I Case II

Figure 8.3: 1-D Consolidation Tests of OC Clays (Case I and Case II)

74
Refresher Class Geotechnical

Case I: ’f  ’c

 For an overconsolidated (OC) soil where the final effective


overburden pressure does not exceed the preconsolidation pressure,
primary consolidation settlement can be calculated using the
recompression index, Cr:

 C   ' f 
S c    r  H log 

 1  eo    'o 

where: Sc = Primary consolidation settlement


 ’o = Initial effective overburden pressure
 ’f = Final effective overburden pressure (’o +)
H = Thickness soil layer
Cr = Recompression index
eo = Initial void ratio

Case II: ’f > ’c

 For an overconsolidated (OC) soil where the final effective


overburden pressure does exceed the preconsolidation pressure,
primary consolidation settlement can be calculated using the
recompression index, Cr and the compression index, Cc:
Δe1 => Δe2 =>
 C  '   C    ' f 
S c    r  H log c    c  H log 

 1  eo  '
 o  1  eo  '
 c  

where: Sc = Primary consolidation settlement


 ’o = Initial effective overburden pressure
 ’f = Final effective overburden pressure (’o +)
H = Thickness soil layer
Cr = Recompression index
Cc = Compression index
eo = Initial void ratio

75
Refresher Class Geotechnical

8.5 Empirical Relationships of Cc and Cr

 The compression and recompression indices can be estimated from


the following empirical relationships.

 For all clays: Cc  1.15(e0  0.35) and Cr  5% to 10% of Cc

where: eo = Initial void ratio

 For undisturbed clay of low to moderate sensitivity:

Cc  0.009( LL  10)

where: LL = Liquid limit

8.6 Rate of Consolidation

 Degree of consolidation U = St/Sc


St = settlement of the layer at time, t
Sc = total settlement

 Degree of consolidation U = (uo – u)/uo


uo = initial excess pore pressure
u = excess pore pressure at time, t

 The average degree of consolidation, U, of a saturated clay layer is


a function of the nondimensional time factor, Tv
cv t
Tv 
H d2
where: Tv = Time factor
cv = coefficient of consolidation
t = time
Hd = length of the drainage path

76
Refresher Class Geotechnical

Hd = H/2 for two-way drainage


= H for one-way drainage
(where H is thickness of the layer)

Figure 8.4: Two-Way and One-Way Drainage Paths

 U = Degree of (%) Consolidation

Table 8.0: Approximate Time Factors


(CERM 2015 Table 40.1)

U Tv
0.10 0.008
0.20 0.031
0.30 0.071
0.40 0.126
0.50 0.197
0.55 0.238
0.60 0.287
0.65 0.340
0.70 0.403
0.75 0.477
0.80 0.567
0.85 0.684
0.90 0.848
0.95 1.129
0.99 1.781
1.00 

 Reference CERM (2018) Chapter 40.8 and Das (2007) Chapter 1.15.

77
Refresher Class Geotechnical

EXAMPLE 8.1
A 15-ft thick clay is bounded by sand at the top and bottom. The clay has
a coefficient of consolidation of 0.3 ft2/day. Determine the time when 50%
and 90% of the total settlement will occur.

Solution:
Double drainage  Hd = _______

From Table 8.0: For U = 50%  Tv = _______

For U = 90%  Tv = _______

Calculate the time for 50% and 90 % of consolidation to occur:

Tv H d2 (_________)(7.5 ft) 2
t50    37 days
cv ft 2
0.3 day

Tv H d2 (_________)(7.5 ft) 2
t90   2
 159 days
cv ft
0.3 day

(Answers given in Appendix A)

78
Refresher Class Geotechnical

9.0 Shallow Spread Foundations

9.1 Shear Strength Parameters

 Shear strength is expressed by the Mohr-Coulomb failure criterion:

𝜏 = 𝑐 + 𝜎 𝑡𝑎𝑛∅

' = shear strength


' = effective stress normal to shearing plane
' = effective internal angle of friction
c' = effective cohesion, or apparent cohesion

 The unconfined compression, direct shear, and triaxial tests are


common methods for obtaining shear strength parameters for soils.

 The unconfined compression test measures c for saturated clay soils.

 The direct shear test is used to measures  and c for sands, silts and
clays. Drainage is not controlled, therefore it is considered an
undrained test.

 The consolidated-undrained (CU) and consolidated-drained (CD)


triaxial tests are more precise, but also more expensive methods for
obtaining shear strength parameters for sands, silts and clays. Note
the CU triaxial test can measure both total  and c and effective ’
and c’ strength parameters with pore pressure measurements.
Drainage is allowed in the CD traixial test so pore pressure due to
shear dissipates, thus only measures effective ’ and c’. The
unconsolidated-undrained (UU) is also used to measure total
stresses.

 Reference CERM (2018) Chapter 35.17 and Das (2007) Chapter 1.15.

[Note: Additional material on shear strength parameters and testing methods


are provided in the supplement for Geotechnical depth students.]

79
Refresher Class Geotechnical

9.2 Types of Foundations

 Foundations can be classified as shallow or deep:

(a) Shallow: spread footings and mats


(b) Deep: Driven piles, drilled shafts, and piers

 For shallow foundation, depth is shallower than its width.

Figure 9.0: Shallow Spread Footing Dimensions

 For deep foundation, depth (Df) is larger than its width (B). Generally,
deep foundations have the ratio (10  Df /B).

 Table 9.0 gives examples of appropriate foundation choices for


various soil conditions (not exhaustive). Also consider that driven
piles may not be appropriate if soil they must penetrate soil layers
with large obstructions (i.e. boulders, junk fill with construction debris,
etc.) that may cause deflection, pile tip damage, or prevent pile from
reaching design embedment depth.

Table 9.0: Foundation Types Correlated with Soil Conditions


(Source: Adapted from McCarthy, 2007)

Appropriate Foundation
Soil Conditions Design Comments
Type and Location
Spread footings most appropriate for
conventional foundation needs. A deep
foundation such as piles could be
required if uplift or other unusual
forces (e.g. seismic, effect of flood)
could act.

80
Refresher Class Geotechnical

Table 9.0 (Cont.): Foundation Types Correlated with Soil Conditions


(Source: Adapted from McCarthy, 2007)

Appropriate Foundation
Soil Conditions Design Comments
Type and Location

Spread footings most appropriate for


conventional foundation needs. Also
see comment for (1) above.

Spread footings would be appropriate


for low to medium range of loads, if
not installed too close to soft clay
layer. If heavy loads are to be carried,
deep foundations might be required.

Spread footing may settle excessively


or require very low bearing pressures.
Consider mat foundation, or ground
improvement (e.g. vibrofloatation).
Driven piles could be used and would
densify the sand.
Spread footing probably not
appropriate. Friction piles or piers
would be satisfactory if some
settlement could be tolerated. Long
piles would reduce settlement
problems. Also consider mat or
floating foundation.

Deep foundation—piles, piers,


caissons—bearing on/in rock

Spread footings in upper sand layer


would probably experience large
settlement because of the underlying
soft clay layer. Consider drilled piers
with a bell formed in hard clay layer,
or other pile foundation into hard clay
layer.

81
Refresher Class Geotechnical

Table 9.0 (Cont.): Foundation Types Correlated with Soil Conditions


(Source: Adapted from McCarthy, 2007)

Appropriate Foundation
Soil Conditions Design Comments
Type and Location

Deep foundation best—cast-in-place


piles such as augercast piles or bulb
piles into sand layer appear most
appropriate.

Deep foundation types extending into


medium dense sand, or preferably,
into compact glacial till. Strong
possibility for drilled pier with bell
constructed in till. Also consider cast-
in-place and driven concrete pile,
timber pile, and pipe pile.

Deep foundations penetrating through


fill are appropriate. With piles or
piers, consider stopping in upper zone
of sand layer so as to limit
compression of clay layer. Also
consider replacing poor fill with
compacted structural fill then using
spread footings in new fill.

If foundation loads are not too heavy,


consider using piles of piers bearing
in the upper zone of sand layer and
check for settlement. If foundation
loads are heavy, consider driven piles
(steel) or caissons to rock.

Foundations should bear directly on


the rock which is relatively close to
ground surface. If no basement areas
are planned, consider piers. If
basement areas are useful, consider
full excavation to rock and
construction of two basement levels.

82
Refresher Class Geotechnical

9.3 Bearing Pressure of Shallow Spread Foundations

 The gross bearing pressure is the total applied pressure of the


foundation acting on the soil at the base of the foundation, including
the weight of the footing (Wc) and overlying soil (Ws).

Pg
Qg  where Pg = Pnet + Wc + Ws
A

where: Qg = Gross bearing pressure


Pg = Gross vertical load
A = Area of applied pressure

 The net bearing pressure is the applied pressure, neglecting the


weight of the footing and overlying soil.
P
Q  net
net A

where: Qnet = Net bearing pressure


Pnet = Net vertical load
A = Area of applied pressure

 The net allowable bearing pressure is the maximum allowable


applied pressure, neglecting the weight of the footing and overlying
soil. The net allowable bearing pressure is determined by dividing the
net bearing capacity (Section 9.4) by an acceptable factor of safety
qnet ( Pnet ) all
(Qnet )all  
FS A

where: (Qnet)all = Net allowable bearing pressure


(Pnet)all = Net allowable vertical load
qnet = Net bearing capacity (see Section 9.4)
FS = Factor of safety (see Section 9.4)
A = Area of applied pressure

83
Refresher Class Geotechnical

9.4 General Bearing Capacity Theory

 The ultimate bearing capacity is theoretically the bearing pressure at


which shear failure will occur.

 Terzaghi’s general bearing capacity equation is given as:

q  cN   D N  0.5 BN
ult c f q 

where: qult = Ultimate bearing capacity


c = cohesion
Df = Depth of footing
 = Unit weight of the soil
B = Width or diameter of footing
Nc, Nq, N = Bearing capacity factors based on 

 The net bearing capacity is corrected for the overburden pressure of


the soil, giving the additional bearing capacity beyond the pressure
applied by the soil. (Note than when the groundwater table is above
the base the footing, use the effective vertical stress, ’D at a depth
of Df.)
q q D
net ult f

 The net allowable bearing capacity is the maximum bearing pressure


the soil can safely support with a reasonable factor of safety (typically
2 to 3 for foundations):

q
(q )  net
net all FS

 Note that bearing capacity and bearing pressure can be thought of in


terms of “supply” and “demand”. The allowable bearing capacity is
the available supply, which must be greater than or equal to the
applied bearing pressure, which is the demand placed on the soil.

84
Refresher Class Geotechnical

q 
(qnet ) all   net    Qnet applied
 FS capacity

and
q 
qall   ult 
 FS capacity
 Qg applied

Note: Qg, Qnet, qult and qnet are defined earlier in this chapter.

 Reference CERM (2018) Ch 36.5 and Das (2007) Chs 3.3 - 3.4.

NOTE: Contribution of bearing capacity factors (Nc, Nq, N) in general


capacity equation

 The Nc term of the general bearing capacity equation is the


contribution of the cohesion of the soil. If the footing bears on a
granular material where cohesion is equal to zero, this term will equal
zero, and thus can be eliminated from the equation.

 The Nq term is the contribution of the overburden pressure (or


surcharge) of the soils from the ground surface to a depth, D, equal
to the depth of the footing. If the groundwater table is below the depth
of the footing, then ’D = D.

 The N term is the contribution of the unit weight of the soils below
the base of the footing. See section 9.7 for details on the effect of
groundwater on bearing capacity.

85
Refresher Class Geotechnical

Table 9.1: Terzaghi’s Bearing Capacity Factors

 Nc Nq N  Nc Nq N
0 5.70 1.00 0.00 26 27.09 14.21 11.23
2 6.30 1.22 0.18 27 29.24 15.90 12.90
4 6.97 1.49 0.38 28 31.61 17.81 14.74
5 7.34 1.64 0.42 29 34.24 19.98 17.11
6 7.73 1.81 0.62 30 37.16 22.46 19.73
8 8.60 2.21 0.90 31 40.41 25.28 23.01
10 9.61 2.69 1.25 32 44.04 28.52 26.84
11 10.16 2.98 1.45 33 48.09 32.23 31.30
12 10.76 3.29 1.67 34 52.64 36.50 36.46
13 11.41 3.63 1.93 35 57.75 41.44 42.43
14 12.11 4.02 2.21 36 63.53 47.16 49.59
15 12.86 4.45 2.54 37 70.01 53.80 58.70
16 13.68 4.92 2.91 38 77.50 61.55 70.07
17 14.60 5.45 3.34 39 85.97 70.61 84.07
18 15.52 6.04 3.82 40 95.66 81.27 100.4
19 16.56 6.70 4.37 41 106.8 93.85 122.4
20 17.69 7.44 4.97 42 119.7 108.8 151.2
21 18.92 8.26 5.75 43 134.6 126.5 188.7
22 20.27 9.19 6.61 44 152.0 147.8 236.5
23 21.75 10.23 7.55 45 172.3 173.3 297.5
24 23.36 11.40 8.58 48 258.3 287.9 780.1
25 25.13 12.72 9.70 50 347.5 415.1 1153

 Reference CERM (2018) Ch 36.5 Table 36.2

86
Refresher Class Geotechnical

IMPORTANT NOTE: The general form of the bearing capacity equation is


only applicable for continuous footings. For other geometries, shape
factors and/or depth factors are applied.

Table 9.2: Shape Factors for Various B/L Values


(Source: CERM Tables 36.4 and 36.5)

B/L Sc S
1 (Square) 1.25 0.85
0.5 1.12 0.90
0.2 1.05 0.95
0.0 (Continuous) 1.00 1.00
1.0 (Circular) 1.20 0.70

 With the shape factors applied, Terzaghi’s bearing capacity equation


is given as:

qult  cNc Sc   D f Nq  0.5 BN S

 Note that Terzaghi’s bearing capacity equation does not have depth
factors applied.

 The bearing capacity can be estimated for rectangular geometries by


interpolating shape factors Sc and Sfrom Table 9.2.

 Note other methods such as Meyerhof or Vesic use different bearing


capacity factors, as well as shape, depth, and inclination factors that
are applied to each term of the general bearing capacity equation.
These methods are outside the scope of this refresher course.

87
Refresher Class Geotechnical

EXAMPLE 9.0
Determine the ultimate and the net allowable bearing capacities for the
continuous footing shown. Use Terzaghi’s method and assume a factor
of safety of 2.5.

Solution:

From Table 9.1: Nc = _________, Nq = _________, N = _________

From Table 9.2: Sc = S = 1.0

1) Solve for the ultimate bearing capacity, qult.

qult  cNc Sc   D f N q  0.5 BN S


qult  (500 lb
)(25.13)(1)  (112 lb
)(2.5ft)(12.72)  0.5(112 lb
)(3.0ft)(9.70)(1)
ft 2 ft 3 ft 3
qult  17,756 psf

2) Solve for the net bearing capacity, qnet.

qnet  qult   D f  17,756 psf  (112 pcf )(2.5 ft)  17,476 psf

3) Solve for the net allowable bearing capacity, (qnet)all.

qnet 17,476 psf


(qnet )all    6990 psf
FS 2.5

(Answers given in Appendix A)

88
Refresher Class Geotechnical

9.5 Bearing Capacity in Clay

 For bearing capacity in cohesive soils, where  = 0, the bearing


capacity factors Nq = 1 and N = 0. Therefore, using Terzaghi's bearing
capacity theory, the ultimate and net bearing capacity equations for
clay soils will simplify as follows:

q ult  cN c S c   D f

qnet  cNc Sc

 The undrained shear strength (su) of clay is equal to the cohesion (c),
which is one-half of the unconfined compressive strength (qu), as
shown:
qu
su  c 
2

 Reference CERM (2018) Ch 36.7.

EXAMPLE 9.1

A 6-ft square footing is founded in a clay soil 3 ft below the ground surface.
The soil parameters are  = 125 pcf, c = 1200 psf, and  = 0. Determine
the net allowable bearing capacity. Assume a factor of safety of 3.

Solution:

From Table 9.1: Nc = 5.70

From Table 9.2: Sc = 1.25

qnet  cN c S c  (1200 psf)(5.70)(1.25)  8550 psf

qnet 8550 psf


(qnet )all    2850psf
FS 3

89
Refresher Class Geotechnical

9.6 Bearing Capacity in Sand

 For bearing capacity in cohesionless soils, where c = 0, the cohesion


term of the bearing capacity equation will be zero. Using Terzaghi's
bearing capacity theory, the ultimate and net bearing capacity
equations for cohesionless soils will simplify as follows:

qult   D f N q  0.5 BN  S 
𝑞 = 𝛾𝐷 𝑁 + 0.5 𝛾𝐵𝑁 𝑆 − 𝛾𝐷
q net   D f ( N q  1)  0.5 BN  S 

 Reference CERM (2018) Ch 36.8.

EXAMPLE 9.2
A 3-ft x 3-ft square footing bears at the surface of a sand deposit for which
 =32 and  = 125 lb/ft3. The water table is very deep. Determine the
gross allowable force (lbs) that this footing can carry. Assume a factor of
safety of 2.0.

Solution:
c = 0,  =32, N = 26.84, S = 0.85

Calculate the ultimate bearing capacity in sand (use bearing capacity


equation, simplified for c = 0):

qult   D f N q  0.5 BN S

qult  0  (0.5)(125 pcf )(3 ft)(26.84)(0.85)  4278psf

Calculate the allowable bearing pressure (ultimate capacity/FS since we


need gross allowable force):

qult (4278 psf )


qall    2,139 psf 
FS 2.0

Calculate the allowable gross vertical force:

Pg
all
= qall A = (2139 psf)(3 ft)2 =19,251 lb

90
Refresher Class Geotechnical

9.7 Effect of Groundwater Table on Bearing Capacity

Case 1: Df  D1  0
 The water table is located at or above the base of the footing.
 The factor “Df” in the “Nq” term is the effective stress, ’D at the base
of the footing:
 ' D  D1   ' D2

 The value of  in the “N” term is taken as:

 '   sat   w

Case 2: B > d > 0 [ d=B ]


 The water table is located between the base of the footing and a
depth of “B” below the base of the footing.
 The value of  in the “N” term is taken as:

d  d
   1   '
B  B
Case 3: d  B
 When the water table is located below the base of the footing by a
depth greater than or equal to “B”, the water table effect is negligible.

 Reference CERM (2018) Ch 36.11 and Das (2007) Chapter 3.5.

NOTE: The effects of groundwater are generally negligible for cohesive


soils ( = 0).

91
Refresher Class Geotechnical

EXAMPLE 9.3
After temporary dewatering, a square footing 5 feet by 5 feet will be
constructed to bear on sand at a depth of 3.5 feet below the ground
surface. Prior to construction, groundwater was at a depth of 2 feet and
is expected to return to that depth when dewatering is terminated. Soil
properties are  = 116 pcf, sat = 120 pcf, ’ = 34 degrees. Use Terzaghi’s
method and a factor of safety of 2. Determine the net allowable soil
bearing pressure for the footing.

Solution:
Nq  36.50,N  36.46, S  0.85

Terzaghi’s simplified bearing capacity equation with shape factor:


qnet   D f ( N q  1)  0.5 BN  S

Calculate net bearing capacity with the groundwater depth at 2.0 feet,
(Case 1):
 'D   D1   ' D2  _______ psf;  '   sat   w  ______ pcf

qnet   'D ( N q  1)  0.5 ' BN S


qnet  (_____)(36.50  1)  0.5(_____)(5 ft)(36.46)(0.85)
qnet  15,766psf

Calculate net allowable bearing capacity:


q 15,766 psf
(qnet )all  net   7883psf
FS 2.0

(Answers given in Appendix A)

92
Refresher Class Geotechnical

9.8 Eccentric Loads on Rectangular Shallow Spread Footings

 Eccentric loading condition is created when footing carries a


moment (M) in addition to a vertical load (P).

Figure 9.1: Eccentrically Loaded Foundations


(Source: Das, 2007, p. 146)

Note: In Figure 9.1, eccentricity is shown in the “B” direction, however it


can occur in the “L” direction, or both. The equations given in this section
should be used for eccentricity in the “B” or “L” direction. Eccentricity in
both directions is beyond the scope of this course.

M
Eccentricity, e 
P

 For eccentricity inside the middle third of the footing (e < B/6):

𝑃 6𝑒 𝑃 6𝑒
𝑞 = 1− 𝑞 = 1+
𝐵𝐿 𝐵 𝐵𝐿 𝐵

93
Refresher Class Geotechnical

 For eccentricity outside the middle third of the footing (e > B/6):

4𝑃
𝑞 <0 𝑞 =
3𝐿(𝐵 − 2𝑒)

 When eccentricity is outside of the middle third of the footing (i.e.


eB>B/6 or eL>L/6), bearing capacity is computed using the effective
footing width:

B '  B  2e B or L'  L  2eL

where: eL = eccentricity along the footing length


eB = eccentricity along the footing width

 To calculate the ultimate bearing capacity, the value of B’ is used in


the general bearing capacity equation when the eccentricity is in the
“B” direction. L’ is used when the eccentricity is in the “L” direction.

 The value of B’ or L’ should be used as applicable to determine the


Terzaghi shape factors as defined in Table 9.2.

 The ultimate, net and allowable bearing capacity equations (from


Terzaghi’s general bearing capacity equation) become:

𝑞 = 𝑐𝑁 𝑆 + 𝛾𝐷 𝑁 + 0.5𝛾𝐵 𝑁 𝑆

q net  q ult  D f

qnet
(qnet )all 
FS

 Reference CERM (2018) Ch 36.12 and Das (2007) Ch 3.11.

94
Refresher Class Geotechnical

EXAMPLE 9.4
A 3 ft x 6 ft rectangular footing has a load, P, of 40,000 lb and a moment,
M, of 25,000ft-lb. The foundation is located with its base 3 ft below the
ground surface and is bearing on sand with an effective internal friction
angle of 30° and a moist unit weight of 118 pcf. Assume the groundwater
is very deep. Determine the following (use a factor of safety of 2):

A. Ultimate bearing capacity of the foundation


B. Net bearing capacity of the foundation
C. Net allowable bearing capacity of the foundation
D. Is the foundation acceptable?

Solution:
Determine eccentricity:
M
eB   ___________ ft
P

Determine effective footing width:


B 3 ft B
  0.5 ft eB  B '  B  2eB  _________ ft
6 6 6

95
Refresher Class Geotechnical

Select bearing capacity factors and shape factors:


Nq  22.46, N  19.73, Interpolate S

Use Table 9.2 to interpolate the shape factor for a rectangular


geometry with B’/L = 0.29

0.95  0.9 0.95  ( S )0.29


Interpolation:   S  0.94
0.2  0.5 0.2  0.29

A. Calculate the ultimate bearing capacity:


qult   D f N q  0.5 B ' N S
qult  (118 pcf )(3ft)(22.46)  (0.5)(118 pcf )(1.75ft)(19.73)(0.94)
qult  9866 psf

B. Calculate the net bearing capacity:


qnet  qult   D f  9866 psf  (118pcf )(3ft)  9512psf

C. Calculate the net allowable bearing capacity:


q 9512 psf
(qnet )all  net   4756psf
FS 2.0

D. Determine the maximum applied pressure, Qmax:


4P 4(40, 000 lb)
Qmax    5079 lb ft 2
3L ( B  2e) 3(6 ft)(3 ft  2(0.625 ft))

Is the footing size appropriate? Qmax > (qnet)all  NO

(Answers given in Appendix A)

96
Refresher Class Geotechnical

10.0 Lateral Earth Pressure and Earth Retention Structures

10.1 Earth Pressure Introduction

 Earth pressure is the force per unit area exerted by soil. The ratio of
horizontal to vertical stress is called coefficient of lateral earth
pressure (K).
'
K h
 'v

 Earth pressure forces can be at-rest (Fig 10.0a), active (10.0b) or


passive (10.0c).

No Wall Movement Outward movement Inward movement


𝝈𝒉 (𝒂𝒕 𝑹𝒆𝒔𝒕; 𝒌𝟎 )  𝝈𝒉 (𝑨𝒄𝒕𝒊𝒗𝒆; 𝒌𝒂 ) 𝝈𝒉 (𝑷𝒂𝒔𝒔𝒊𝒗𝒆; 𝒌𝒑 )



 45 + φ/2 45 - φ/2

(a) At-rest (k0) (b) Active Case (ka) (c) Passive Case (kP)

Figure 10.0: Nature of Lateral Earth Pressure on a Retaining Wall

Table 10.0 Typical range of lateral earth pressure coefficients


(Source: CERM Table 37.2)

Condition Granular Soil Cohesive Soil


Active 0.20 - 0.33 0.25 - 0.5
Passive 3-5 2-4
At-Rest 0.4 - 0.6 0.4 - 0.8

97
Refresher Class Geotechnical

10.1.1 At-Rest Earth Pressure

 Under conditions of zero horizontal displacement, the soil is said to


be at-rest: Ko  1- sin ’ (granular soils)

 Reference CERM (2018) Ch 37.7.

10.1.2 Active Earth Pressure

 For a level backfill ( =0), the following equation is used to determine


the active earth pressure (pa) for all types of soils.

pa  Ka H  2c Ka

For saturated clay soils,  = 0; Ka = 1: pa   H  2c

For granular soils, c = 0: pa  K a H

 The critical depth, zcr, which is the depth at which the horizontal
pressure is zero, can estimated by:

2𝑐
𝑍 =
𝛾 𝑘

 The total active resultant force (without surcharge or cohesion) is


solved for by:

1 1
Ra  pa H  K a H 2
2 2

 Reference CERM (2018) Ch 37.5 and Das (2007) Ch 7.3.

98
Refresher Class Geotechnical

10.1.3 Passive Earth Pressure

 For a level backfill ( =0), the following equation is used to determine


the passive earth pressure for all types of soils.

p p  K p H  2c K p

For saturated clay soils,  = 0, Kp = 1: p p   H  2c

For granular soils, c = 0: p p  K p H

 The total passive resultant force (without surcharge or cohesion) is


solved for by:

1 1
Rp  p p H  K p H 2
2 2

 Reference CERM (2018) Ch 37.6 and Das (2007) Ch 7.10.

99
Refresher Class Geotechnical

10.2 Rankine’s Earth Pressure Theory

 Neglects friction between back of wall face and backfill soil (   0 ).

 For sloping backfill:

 cos   cos 2   cos 2  


K a  cos   
 2 2 
 cos   cos   cos  

 cos   cos 2   cos 2  


K p  cos   
 2 2 
 cos   cos   cos  
where:
 = Friction between back of wall face and backfill soil
(neglected in Rankine earth pressure theory)
 = Internal angle of friction
 = Angle of sloping backfill

 For level backfill ( = 0):

1 1  sin 
Ka    tan2 (45  )
K p 1  sin 2

1 1  sin 
Kp    tan 2 (45  )
K a 1  sin 2

 Reference CERM (2018) Chs 37-5 to 37-6 and Das* (2007) Chs 7.4
& 7.11 (*includes design tables of Rankine lateral earth pressure
coefficients).

100
Refresher Class Geotechnical

10.3 General Earth Pressure Diagrams

CASE 1 CASE 2
Active Pressure Active Pressure with Surcharge

 H 2K a H  H 3K a
Case 1: Ra  M  Ra 
2 3 6
 H 2K a q s H K a  H 3K a
2
Case 2: R a  qs HK a  M  Ray  
2 2 6

CASE 3
Active Pressure with Groundwater

 H12K a  'H 22K a  w H 22


Ra    H1H 2K a  
2 2 2
2 2 3 3
 H1 K a  H1   H1H 2 K a  'H 2K a  w H 2
M  Ra y    H2    
2  3  2 6 6

101
Refresher Class Geotechnical

CASE 4
Two Soil Layers, (Top Layer Weaker) Two Soil Layers, (Top Layer Stronger)

1H12 Ka1  2 H 22 Ka 2
Ra    1H1H 2 K a 2 
2 2
 1H12 K a1  H1 2 3
  1H1H 2 K a 2  2 H 2 K a 2
M  Ra y    H2   
2  3  2 6

CASE 5
Two Soil Layers with Surcharge, Two Soil Layers with Surcharge,
Top Layer Weaker Top Layer Stronger

1H12 Ka1  2 H22 Ka 2


Ra  qs H1Ka1   (qs  1H1) H2 Ka 2 
2 2
2 2 3
 H1  1H1 Ka1  H1  (qs  1H1) H2 Ka 2  2 H 2 Ka 2
M  Ra y  qs H1Ka1   H2     H2   
 2  2  3  2 6

102
Refresher Class Geotechnical

CASE 6 CASE 7
Active Pressure with Cohesion Passive Pressure with Cohesion

 H 2 Ka
Case 6: Ra   2cH Ka
2
 H 2K p
Case 7: Rp   2cH K p
2

10.4 Coulomb’s Earth Pressure Theory

 Includes friction () between back of the wall face and backfill soil.

sin 2 (   )
Ka 
2
2  sin(   ) sin(   ) 
sin  sin(   )1  

 sin(   ) sin(   ) 

sin 2 (   )
Kp  2
2  sin(    ) sin(    ) 
sin  sin(    ) 1  
 sin(    ) sin(    ) 

103
Refresher Class Geotechnical

where:  = Friction between back of wall face and backfill soil


(see CERM 2018 Table 37.1 for values)
 = Internal angle of friction
 = Angle of sloping backfill
 = Inclination of the wall face on the back side

 Note that Coulomb’s equation already adjusts for the sloped backfill,
therefore the vertical and horizontal components are given as:

( R a ) v  R a sin(90     )

( R a ) h  R a cos( 90     )

 Reference CERM (2018) Ch 37.3, and Das* (2007) Chs 7.5 & 7.12
(*includes design tables of Coulomb lateral earth pressure
coefficients)

10.5 Rigid Retaining Walls

 Retaining walls are designed by trial and error adjustment of wall


dimensions using earth pressure theory.

 The wall must be safe against sliding, overturning, bearing failure,


global failure, and must be economical.

 Rely on flexural strength, the weight of the wall, and the retained
backfill soil to resist active earth pressure.

 Rankine or Coulomb theories can be used (Rankine is easiest).

 Consider all forces: active, passive, weight of wall, groundwater and


surcharge.

104
Refresher Class Geotechnical

 Gravity retaining walls are generally constructed with plain concrete


or stone masonry, and depend primarily on their own weight for
stability.

𝛽 𝛽

Figure 10.1: Gravity Retaining Walls (a) Rankine and (b) Coulomb
(Source: adapted from Das, 2007)

 Cantilever retaining walls are generally made of reinforced concrete


that consists of a thin stem, a base slab, and occasionally an optional
key. They depend primarily on the weight of retained soil above the
base slab for stability.

𝑯
𝑯
Ra
Ws
𝛽
Wc

𝐻
3

Figure 10.2: Cantilever Retaining Wall (Rankine)

105
Refresher Class Geotechnical

EXAMPLE 10.0
A 10 ft high gravity retaining wall with flat backfill ( = 0) retains a clean
sand for which  = 120 lb/ft3 and  = 32. Using Rankine’s earth pressure
theory, calculate the total active earth pressure, the active resultant, and
the overturning moment about the toe.

Solution:

See lateral earth pressure diagram:


(Case 1)

Calculate the active earth pressure coefficient:

 
K a  tan 2 (45  2 )  tan 2 (45  322 )  0.307

Calculate the active earth pressure and resultant force:

p a   HK a  (120 pcf )(10 ft )(0.307 )  368 psf

1 1
Ra  pa H  (368 psf )(10 ft)  1842 plf
2 2

Calculate the overturning moment:

H   10 ft 
M  Ra    1842 plf    6140
ft-lb
ft
 3   3 

106
Refresher Class Geotechnical

10.6 Stability of Rigid Retaining Walls

 Overturning about toe:


M R
FSOT 
M 0

MR = moments that resist overturning


Mo = moments that cause overturning

 Sliding:
FR
FS SL 
F0

FR = resisting forces
Fo = driving forces

 Notes for analysis of cantilever retaining walls:

1. FS against overturning (moments taken about the toe) should be


greater than 1.5 if passive resistance is neglected and greater
than 2.0 if passive resistance is included.

2. FS against overturning can be increased by increasing the width


of the base (B).

3. FS against sliding should be greater than 1.5 for granular


foundation soils and greater than 2.0 for cohesive foundation
soils

4. FS against sliding can be increase by increasing the width of the


base (B), or by adding a key to the base of the wall.

5. Passive resistance may not be reliable and is often neglected in


stability analysis.

107
Refresher Class Geotechnical

6. The weight of the soil over the toe of the wall is generally very
small and often neglected when computing the total weight.

7. The unit weight of concrete is generally taken as 150 pcf.

8. Reference CERM (2018) Ch 37.12.

CASE 8
Cantilever Retaining Wall with Flat Backfill,  =0 (Rankine)

Driving: Resisting:
2
1H Ka  2D2K p
Ra  Rp   Fs
2 2

H  D
M  Ra   M  W (x )  Rp  
 3  3

108
Refresher Class Geotechnical

CASE 9
Cantilever Retaining Wall with Sloping Backfill (Rankine)

Driving: Resisting:

  ( H ') 2 K a   2D2K p
Ra , h  Ra cos    1  cos  Rp   Fs
 2 2
 

 H ' D
M  Ra , h   M  W ( x )  Ra , v  B   R p  
 3   3

Note: For Cases 8 and 9, reference CERM Table 38.3 for values of cA (using
values for concrete). Alternatively, cA can be taken as 23 c .

cA = 0 in granular soils where c = 0.

109
Refresher Class Geotechnical

EXAMPLE 10.1
Use Rankine’s earth pressure theory to check the stability (FOS against
sliding and overturning) of the retaining wall shown in the figure below.
The unit weight of concrete is 150 pcf. The water table is well below the
bottom of the footing and does not affect the bearing capacity. Neglect
passive pressure and weight of the backfill over the toe.

Solution:
Determine the resultant active earth force, Ra:
30
K a  tan 2 (45  )  0.333
2
 H 2 Ka (120 pcf )(8 ft)2 (0.333)
Ra    1279 plf
2 2

Determine sum of the vertical forces and the moment about the toe
(complete the table below):

W (lb/ft) Arm (ft) MR (ft-lb/ft)


1
2
3
V  7440plf M R  38,940ft  lb / ft

110
Refresher Class Geotechnical

For factor of safety against sliding:

Fs  V tan   (7440 plf ) tan 20   2708 plf

FR Fs 2708 plf


( FS ) slide     2.12
FO Ra 1279 plf

For factor of safety against overturning:

M R M R 38,940 ft-lb
( FS )OT    ft
 11.4
M O Ra  3
H  8 ft 
(1279 lbft )  
 3 
(Answers given in Appendix A)

10.13 Equivalent Fluid Density

It is convenient to express the lateral pressures acting on retaining walls in


terms of an equivalent fluid density (eq). This simplifies calculations and
relieves the structural engineer of having to estimate the weight of the backfill
and retained soils to interpret the lateral earth pressure for design.

For drained backfill:


Free draining granular backfill typically weighs less than the retained soil.
The equivalent fluid density should be calculated based on the weight of the
granular backfill in the entire Rankine zone (to 45 + /2) behind the wall. If
entire Rankine zone is not covered by granular fill, the weight used to
calculate the equivalent fluid density should be increased in proportion to the
size and shape of the backfill zone. The equivalent fluid density is given by:

 eq  k t

Where:
t = the total (moist) unit weight of the backfill or retained soil
whichever controls.
k= ko for at-rest pressure, ka for active pressure, or kp for passive pressure.
For saturated backfill:

111
Refresher Class Geotechnical

When groundwater rises behind a retaining wall, the backfill remains


saturated. The effect of the groundwater must be added. For saturated
backfill, the equivalent fluid density is given by:

 eq  k sat  1  k   w

Where:
sat = the saturated unit weight of the soil.

For most soils the EFD is in the range: 30 pcf < EFD < 100 pcf.

Reference CERM (2018) Ch 37.11

EXAMPLE 10.2
Soil used to backfill a cantilever retaining wall has a friction angle of 30
degrees and the unit weights, t = 115 pcf and sat = 118 pcf. Determine the
Rankine active equivalent fluid density (EFD) for drained and submerged
conditions.

Solution:
The Rankine active earth pressure coefficient is:

   30 
ka  tan 2  45    tan 2  45    0.333
 2  2
EFD for the Drained Condition:

 eq  ka t  0.333 115 pcf  38 pcf

EFD for the Saturated Condition:

 eq  ka sat  1  ka   w  0.333118 pcf  1  0.333  62.4 pcf  81 pcf

In recommendations, these values would be rounded to 40 pcf and 80 pcf.

112
Refresher Class Geotechnical

11.0 Slope Stability and Braced Excavations

11.1 Slope Stability

 Slope stability is the potential for a slope to withstand movement


under given loading conditions. Stability is determined by
balancing the “Shear Stress” induced along any potential failure
surface under the given loading conditions to available “Shear
Strength” of the soil along that failure surface.

FS = Shear Strength /Shear Stress

 Increase in Shear Stress can be caused by structural loading,


lateral pressure, and transient forces.

 Decrease in Shear Strength may be due to pore water pressure,


weathering effects, and organic materials.

11.1.1 Taylor’s Slope Stability Chart

The slope stability chart shown in Figure 11.0 is the Taylor’s chart
modified by Janbu. This chart applies to slopes of homogeneous
saturated clay (∅ = 0).

Assumptions made are:


 No water is outside of the slope (OR slope maybe fully submerged)
 No surcharge or external loads on slope and no tension cracks
 Shear strength is from cohesion only and remains constant with depth
 The slope failure is rotational and occurs along a circular arc

The factor of safety is given by:


𝑁𝐶
𝐹𝑆 =
𝛾𝐻

where: No = Stability number


𝛾 = Total unit weight of soil (Use 𝛾 for submerge condition)
C = Cohesion (undrained shear strength) of the soil
H = Height of the slope

113
Refresher Class Geotechnical

Figure 11.0: Taylor’s chart modified by Janbu for saturated clay slopes
(Source: CERM 2018)

The stability number is obtained from the chart for a depth factor, d,
and slope angle, β, relative to horizontal. D and H are defined in Figure
11.0 for base, toe and slope failures.
D
d
H

For slope angles greater than about β = 53° the failure surface will pass
through the toe. For steeper slopes extend the chart linearly to No =
3.83 for 𝛽= 90 deg.

The minimum acceptable factor of safety is generally 1.3 to 1.5.

Reference CERM (2018) Chapter 40.10 (pgs 40-7 to 40-8).

114
Refresher Class Geotechnical

EXAMPLE 11.0

A temporary cut is proposed as shown. The cut will be made in clay soil
with an undrained shear strength, c = 625 psf, and 𝛾 = 125 pcf. If the
factor of safety must be at least 1.5, what is the maximum slope angle of
the proposed cut? Use Taylor’s Chart.

Solution:
N oc (FS) H 1.5(125 pcf )(20 ft)
FS   No    6.0
H c 625 psf

D 8
d   0.4
H 20

From Taylor’s Slope Stability

  35 degrees

115
Refresher Class Geotechnical

11.1.2 Method of Slices


In slope stability analysis it is mathematically convenient to divide the
failure mass into a series of vertical slices and to write equations based
on the forces acting on the individual slices. The factor of safety is then
determined by summation.

Figure 11.1: Slope with an arbitrary failure surface divided into slices, and
forces acting on an individual slice.

Forces on the individual slices include:

1. Weight of the soil mass (W),


2. Pore pressures (u), and
3. Forces acting on the sides and bottom of the slice.

The forces are resolved into components normal and parallel to the
bottom of the slice.

Assumptions made for calculation are:

(i) Side forces on each slice are equal & opposite and neglected
(ii) Bottom surface of the circular slice is straight.

For effective stress analysis the factor of safety is given by:


cl  W cos   ul  tan  

F  
W sin 

116
Refresher Class Geotechnical

Where c' and ∅' are the effective cohesion and friction angle of the soil,
W is the slice weight, u is the average pore pressure acting on the bottom
of the slice, l is the length of the bottom of the slice, and 𝛼 is the angle of
the bottom of the slice measured from horizontal.

Figure 11.2: Slope with arbitrary failure surface for nonuniform soil
divided into slices.

For total stress analysis the factor of safety is given by:

F  
cT l  W cos  tan T 

W sin 
Where cT and ϕT are the total stress cohesion and friction angle of the soil
and the other parameters are as previously defined.

For Dry Cohesionless Soil:


𝑡𝑎𝑛∅
𝐹𝑆 =
𝑡𝑎𝑛𝛼 ∅𝑻

α
Where:
α
FS = Factor of Safety
∅ = Internal friction angle Fig. 11.3 Typical slope for
𝛼 = Angle of slope from horizontal line Cohesionless Soil (Sand)

117
Refresher Class Geotechnical

11.2 Braced Excavations

 For temporary construction of trenches, support may be provided by


sheeting, which in turn will be supported by struts braced across the
trench.

 Bracing of sheeting will restrain the movement of the top of sheeting


away from the soil.

 Reference CERM (2018) Chs 39.1 to 39.6 and NAVFAC DM 7.02.

 A summary of walled excavation types is shown in Figure 11.0

Figure 11.4: Support Systems for Walled-Excavation


(Source: NAVFAC DM 7.02, 1986, pg 15)

118
Refresher Class Geotechnical

 Design consideratios for selecting a support system is given in Table


11.0

Table 11.0: Factors in Selecting of Support Systems for Deep Excavations


(Source: NAVFAC DM 7.02, 1986, pgs 16)

Requirements Lends Itself to Use of Comments


Tiebacks or rakers or
Open excavation cantilever walls -
area (shallow excavation)
Soldier pile or
sheetpile walls;
Low Initial Cost -
combined soil slope
with wall
Use as part of Diaphragm (See DM
Diaphragm wall most common
permanent 7.3 Chapter 3) or
as permanent wall.
structure cylinder pile walls
Deep, soft clay Structural or raker
Tieback capacity not adequate
subsurface supported diaphragm
in soft clays.
conditions or cylinder pile walls
Dense, gravelly Soldier pile, diaphragm Sheetpiles may lose interlock on
sand or clay or cylinder pile hard driving.
soils
High in situ lateral stresses are
Deep, Struts, long tiebacks or relieved in overconsolidated
overconsolidated combination of soils. Lateral movements may
clays tiebacks and struts be large and extend deep into
soil.
Diaphragm walls,
Avoid possibly sheetpile Soldier pile wall is pervious
dewatering walls in soft soils
Minimum High preloads on stiff Analyze for stability of bottom of
movements strutted or tie-back wall excavation
Wide excavation Tiebacks preferable except in
(greater than 65 Tiebacks or rakers
very soft clay soils.
feet wide)
Struts more economical but
Narrow Crosslot struts tieback still may be preferred to
excavation (less keep excavation open.

119
Refresher Class Geotechnical

than 65 feet
wide)

11.2.1 Pressure Diagrams for Cross-Braced Excavations

 Due to the bracing, triangular pressure diagram does not develop.

 The following earth pressure diagrams were developed by Peck,


based on measurements of a number of excavations.

Figure 11.5: Apparent pressure diagrams on braced vertical sheeting


(Source: CERM, 2018)

Table 11.1 Computation of Pmax by soil type


(Reference CERM (2018) Chs 39.3 through 39.6)

General Soil Type Criteria Pmax


Sanda Assume c = 0 pmax = 0.65HKa
H
Stiff Clayb 4 pmax = 0.3H
c
H
Soft Clay 6 8 pmax = H - 4c
c
H
Medium Clayc 4 6 --
c
aUse Rankine lateral earth pressure coefficient: K a  tan 2 (45   )
2

120
Refresher Class Geotechnical

bThis is the midpoint of the range pmax = 0.2H to 0.4H; if the movement is minimal
or if the construction duration is short, use pmax = 0.2H
cCompute pmax for soft and stiff condition and use conservative (higher) value

EXAMPLE 11.1
A braced excavation in clay is 10 ft deep. Shear strength of the clay is
250 psf and its unit weight is 120 pcf. Determine the design pressure
diagram and calculate the total force.

Solution:
H (120 pcf )(10 ft )
Check soil type criteria (see Table 11.1):   4.8
c 250 psf

Clay is “medium”, therefore both soft and stiff cases must be evaluated.

Calculate pmax and resultant force, R, for the soft clay case:

p max   H  4 c  (120 pcf )(10ft)  4(250 psf )  200 psf

1 
R   (200 psf )(2.5 ft)   (200 psf )(7.5 ft)   1750 plf
2 

Calculate pmax and resultant force, R, for the stiff clay case (use average
of range):

p max  0.3 H  0.3(120 pcf )(10 ft)  360 psf

1 1
𝑅= (360 psf)(2.5 ft) + [(360 psf)(5)] + (360 psf)(2.5 ft) = 2700 plf
2 2

121
Refresher Class Geotechnical

The total resultant force  R = 2700 plf

11.2.2 Analysis of Cross-Braced Excavations

 Some practitioners use the reaction of an inner support of a continuous


beam (load x span) multiplied by a coefficient of 1.1 as an
approximation of the load carried by each strut. Loads carried by
individual struts can be estimated by

P = 1.1 x Tributary Area


= 1.1 x (pmax x Span)

 There are other approximation methods (i.e moment-hinge method


– see Appendix B for an alternate solution using this method),
however problems on exam should be able to be answered
correctly using any reasonable approximation.

EXAMPLE 11.2
A 30 foot deep excavation is made in a sand profile. A braced cut will be
made as shown in the figure below. The sand has an internal friction angle
of 36 degrees (Ka = 0.260) and unit weight of 122 pcf. Calculate the strut
loads and the lateral force that must be resisted by the soil against the
sheeting below the bottom of the excavation.

122
Refresher Class Geotechnical

Solution:

Calculate the resultant force for each strut and the lateral force that must
be resisted by the soil against the sheeting below the bottom of the
excavation:

pmax  0.65 HK a  0.65(122 pcf )(30 ft)(0.260)  618.5 psf


P1  1.1  p max  H1  1.1 618.5 lb
ft 2
  4ft  10ft2   6123 lb
ft

P2  1.1  p max  H 2  1.1 618.5 lb


ft 2
  10ft2  10ft2   6804 lb
ft

P3  1.1  p max  H 3  1.1 618.5 lb


ft 2
  10ft2  6ft2   5443 lb
ft

P4  1.1  p max  H 4  1.1 618.5 lb


ft 2
  6ft2   2041lb
ft

(Alternate solution given in Appendix B)

123
Refresher Class – Appendix A Geotechnical

APPENDIX A
EXAMPLE PROBLEM ANSWER KEY
EXAMPLE 1.0
120 lb3
Wt 36.0lb lb  ft  101.7 lb
    120 3 ,  d  
Vt 0.3ft 3 ft 1  w 1  0.18 ft 3

Gs w (2.67)(62.4 lb3 )


ft e 0.64
e 1   1  0.64 , n    0.39
d (101.7 lb3 ) 1  e 1  0.64
ft EX2.4 Sample A
wGs (0.18)(2.67)
S 100%  100%  75%
e 0.64

EXAMPLE 2.1
D60 = 0.850 mm, D50 = 0.600 mm D30 = 0.250 mm, D10 = 0.075 mm,
Cu = 11.33, Cz = 0.98

EXAMPLE 2.2
Sample A: PI = NP, GI = 0, “A-1-b (0)” EX2.4 Sample B
Sample B: PI = 11, GI = PGI = (0.01)(30-15)(11-10) = 0.15, “A-2-6 (0)”
Sample C: PI = 31, GI = (87-35)[0.2+0.005(71-40)]+0.01(87-15)(31-10) = 33.58, “A-7-5 (34)”

EXAMPLE 2.3
Sample A: Gravel = 0%, Sand = 13%, Fines = 87%, “MH”
Sample B: Gravel = 2%, Sand = 75%, Fines = 23%, “SC-SM”
Sample C: Gravel = 23%, Sand = 67%, Fines = 10%, “SP-SC”

EXAMPLE 2.4
Sample A: Sand = 15%, Silt = 25%, Clay = 60% USDA = “CLAY”
Sample B: Coarse fraction = 15%; Sand = 17% Normalized = 20%; Silt = 47%, Normalized = 55%; Clay =
21%, Normalized = 25%; USDA = “SILT LOAM”

EXAMPLE 3.1
Soft (d), 35 (b), 0.75 (b), fair (b)

EXAMPLE 4.0

Volume of Wt. of Moist Soil Moist Unit Moisture Dry Unit Weight
Mold (lb) Weight Content (%) (lb/ft3)
(ft3) (lb/ft3)
1/30 3.53 105.9 11 95.41
1/30 3.85 115.5 13 102.2
1/30 4.01 120.3 15 104.6
1/30 3.97 119.1 17 101.8
1/30 3.77 113.1 19 95.04
1/30 3.69 110.7 21 91.49

(d)max  105 pcf, wopt  15 %

I
Refresher Class – Appendix A Geotechnical

EXAMPLE 5.0
hent =5ft+1ft=6ft; hext=2ft+1ft=3ft
Point A: hA=6ft-1.0(2ft)=4ft; hp=4ft-3ft=1ft

EXAMPLE 5.3
Nf = 4, Nd = 8, Total head loss = 12ft,
Head loss per drop = 1.5 ft,
hA = 3.0 ft, hB = hC = 7.5 ft,
Q = 0.0984 ft3/min

EXAMPLE 6.0
Case 1: u=(62.4pcf)(15ft)=936psf
Case 2: u=(62.4pcf)(25ft)=1560psf

EXAMPLE 7.1
I = 0.120

EXAMPLE 7.2
I = 0.7 Example 7.0 Graphical Solution

EXAMPLE 8.1
Hd = H/2 = 7.5 ft,
(Tv)50 = 0.197, (Tv)90 = 0.848

EXAMPLE 9.0
Nc = 25.13, Nq = 12.72, N = 9.70

EXAMPLE 9.3
σ’D = 318.4 psf, ’ = 57.6 pcf

EXAMPLE 9.4
eB = 0.625 ft, B’ = 1.75 ft

EXAMPLE 10.1
W (lb/ft) Arm (ft) MR (ft-lb/ft)
1 6ft  7ft 120pcf  5040 6ft 5040plf  6ft  30,240
2 1ft  7ft  150pcf  1050 2.5ft 1050plf  2.5ft  2625
3 9ft  1ft  150pcf  1350 4.5ft 1350plf  4.5ft  6075
V  7440plf M  38,940ft  lb / ft

II
Refresher Class – Appendix A Geotechnical

EXTENDED SOLUTIONS FOR SELECT PROBLEMS


Example 2.2:

Sample A Summary: F200 is less than 35% and PI is less than 6, therefore use Table 2.1 starting
at far left column (Step 2). Assume GI = 0 (Step 6).

Sample B Summary: F200 is less than 35% and PI is greater than 6, therefore use Figure 2.3 to
plot LL vs. PI (Step 3). Compute PGI (Step 6).

III
Refresher Class – Appendix A Geotechnical

Sample B Solution using Table 2.1:

Sample C Summary: F200 is greater than 35%, therefore use Figure 2.3 to plot LL vs. PI (Step
4). Compute GI (Step 5).

IV
Refresher Class – Appendix A Geotechnical

Sample C Solution using Table 2.1:

V
Refresher Class – Appendix B Geotechnical

APPENDIX B

ALTERNATE SOLUTIONS

CH1: EX 1.0 Alternate Solution

Complete phase diagram:

V (ft3) W (lb)
A 0
W
S
Vt = 0.3 Wt = 36

Compute the weight of solids:


W s  W t  W w  36 lb  W w

Compute the weight of water:


Ws  36  0.18Ws
W w  w W s  0.18W s therefore:
Ws  30.5 lb and Ww  5.5 lb

Compute the volume of water:


W 5.5 lb
Vw  w   0.088 ft 3
 w 62.4 pcf

Compute the volume of solids:


W Ws 30.5 lb
Vs  s    0.183 ft 3
 s Gs w (2.67)(62.4 pcf )

Compute the volume of voids:


Vv  Vt  Vs  0.3 ft 3  0.183 ft 3  0.117 ft 3

VI
Refresher Class – Appendix B Geotechnical

Solve for required parameters:

V (ft3) W (lb)
Vv = 0.117 A 0
0.088 W 5.5
0.183 S 30.5
Vt = 0.3 Wt = 36

Wt 36 lb
Moist unit weight:   3
 120 lb3
Vt 0.3 ft ft

W 30.5 lb
Dry unit weight: d  s  3
 101.7 lb3
Vt 0.3 ft ft

Vv 0.117 ft 3
Void ratio: e   0.64
Vs 0.183 ft 3
Vv 0.117 ft 3
Porosity: n   0.39
Vt 0.3 ft 3
Vw 0.088 ft 3
Degree of Saturation: S    0.75 (75%)
Vv 0.117 ft 3

VII
Refresher Class – Appendix B Geotechnical

CH5: EX 5.0 Alternate Solution

Considering the direction of flow, Point A is 2/3 of distance from the top of
the sand column (entrance) to the bottom (exit), instead of at the center.
Therefore, by interpolation, the total head at Point A is equal to the total head
at the top (entrance) of the sand column minus 2/3(h).

hA  hENT  32 h( Ent to Ext )  6 ft  32 (3 ft)  4 ft

Another way to visualize this problem is to picture piezometers at the top,


bottom and at Point A in the sand column. By spacing the piezometers on a
1:1 vertical to horizontal ratio, the linear hydraulic gradient is clearly visible
with flow in the downward direction.

VIII
Refresher Class – Appendix B Geotechnical

CH11: EX 11.1 Alternate Solution (Moment-Hinge Method):

Calculate pmax:
Ka  tan2 (45  36
2
)  0.260
p max  0.65 HK a  (0.65)(122pcf )(30ft)(0.260)  618.5 psf

The force in the top strut (P1) is solved for by assuming a hinge at the
second strut and summing the moment about the hinge:


P1 (10ft)  618.5
 14 
lb
 14ft  ft 
ft 2
 2 

  14 
618.5 lb2  14ft  ft 
ft 
 2   6061 lb
P1  ft
10ft

The force in the middle strut (P2) is solved for by assuming a hinge at
the third strut and summing the moment about the hinge:

 P2  10ft    6061 lbft  20ft    618.5 lb


ft 2
  24 
 24ft  ft 
 2 

 618.5 lb
ft 2 

 24 
 24ft  ft    6061 lb
2  ft
 20ft 
P2   5691 lb
ft
10ft

IX
Refresher Class – Appendix B Geotechnical

The soil force (P4) is solved for by assuming a hinge at the third strut and
summing the moment about the hinge:

 P4  6ft   618.5 6 
lb

 6ft  ft 
ft 2
2 

 6 
618.5 lb2  6ft  ft 
ft 
 2   1856 lb
P4  ft
6ft

The force in the third strut (P3) is solved for by summing the horizontal
forces of the system:

6061 lb
ft
 5691 lb
ft
 P3  1856 lb
ft
 618.5  lb
ft 2
 30ft 
 ft

P3  618.5 lb2  30ft  6061 lb
ft
 5691 lb
ft
 1856 lb
ft
 4947 lb
ft

X
Refresher Class – Appendix C Geotechnical

APPENDIX C

AT HOME STUDY PROBLEMS

P1 – Chapter 1 (DEPTH PROBLEM)


A soil sample from a 6-ft thick sand deposit was taken to the laboratory
for testing. The sample has a maximum and minimum void ratio of 0.86
and 0.45, respectively, a specific gravity of 2.71 and a moisture content
of 10 percent. The measured in-situ relative density of the soil is 65%.
The decrease in layer thickness in inches to achieve a relative density of
75% is most nearly:

A) 1.6
B) 1.7
C) 1.8
D) 1.9

P1 Solution:
Determine the relationship between the change in height and change in
relative density:

H Vv Vv e H e
   n   
H V V 1 e H 1 e

Compute the in-situ void ratio for 65% and 75%:

e  e m ax  D r (e m ax  e m in )
e 65  0 .86  0.65(0 .8 6  0 .45)  0.5 9
e 75  0 .86  0.75(0 .8 6  0 .45)  0.5 5
 e  0.59  0.55  0.04

Compute the change in height in inches due to the change in void ratio:

H e He (6ft)(0.04)  12in 


  H      1.81in
H 1 e 1 e 1  0.59  ft 

The answer is (C)

XI
Refresher Class – Appendix C Geotechnical

P2 – Chapter 8
A tall building is supported on a large mat foundation with a uniform
pressure of 1000 psf. The mat bears at the surface on silty clay soil 10
feet above the water table. The silty clay soil layer is 28 feet thick and it
is underlain by gravel. The soil below the mat was fully consolidated
before drilling water wells, which lowered the water table to 18 feet below
the foundation. Which, if any, of the layers shown below (A, B or C) will
consolidate after the water table is lowered?

A) Layers A, B, and C will consolidate.


B) Layers B and C will consolidate.
C) Only layer B will consolidate.
D) None of the above.

XII
Refresher Class – Appendix C Geotechnical

P2 - Solution:

Hint: Consolidation will only occur in layers with a higher vertical effective
stress after the change in water table elevation:

The vertical effective stress in the layer A (0 to 10 ft) will not change once
the water table is lowered, therefore there will be no consolidation.

The vertical effective stress in layer B (10 to 18 ft) will increase due to the
decrease in pore water pressure, therefore consolidation will occur.

The vertical effective stress in layer C (18 to 28 ft) will increase due to the
increased vertical effective stress in the layer above it, therefore
consolidation will occur.

The answer is (B)


-------------------------------------------------------------------------------------------

Calculate Effective Stress for INITIAL & FINAL Conditions


(Neglect MAT q=1000 psf for ease of calculation as it remains CONSTANT Before & After Drilling)

Effective Stress BEFORE drilling water wells:


Bottom of Layer A = 100 x 10 = 1000 psf
Bottom of Layer B = 1000 + (120 – 62.4) x 8 = 1461 psf
Bottom of Layer C = 1461 + (120 - 62.4) x 10 = 2037 psf

Effective Stress AFTER drilling water wells:


Bottom of Layer A = 100 x 10 = 1000 psf
Bottom of Layer B = 1000 + 100 x 8 = 1800 psf
Bottom of Layer C = 1800 + (120 - 62.4) x 10 = 2376 psf

Increase in Effective Stress

Layer A = 0
Layer B = 1800 – 1461 = 339 psf (Consolidation will occur)
Layer C = 2376 – 2037 = 339 psf (Consolidation will occur)

XIII
Refresher Class – Appendix C Geotechnical

P3 – Chapter 9
A shallow 5 ft x 5 ft square foundation bears in clay soil with qu = 1.75 tsf at
a depth of 2.5 ft. The footing is designed for a net column load of 100 kips.
Using Terzaghi’s bearing capacity theory, the factor of safety of this footing
is most nearly:

A) 2.0
B) 2.5
C) 3.0
D) 6.0

P3 Solution:
Determine the net bearing capacity (see CH 9.5 for eqn):

( qnet )cap  cN c Sc  (1750 psf )(5.70)(1.25)  12, 469 psf

Determine the net applied load (see CH 9.3 for eqn):

P 100,000 lb
(Qnet )app  net   4,000 psf
A (5 ft)2

Determine the net applied load (see CH 9.3 for eqn):

(qnet )cap 12, 469 psf


FS    3.1
(Qnet ) app 4,000 psf

The answer is (C)

XIV
Refresher Class – Appendix C Geotechnical

P4 – Chapter 9
A net load of 175 kips is applied to a square footing bearing in a sand deposit
at a depth of 3 feet below the ground surface. The sand soil has internal
angle of friction of 35 degrees, unit weight of 125 pcf. The groundwater table
is far below the base of the footing. Using Terzaghi’s method and an
assumed factor of safety against bearing failure of 2, the minimum required
footing width is most nearly:

A) 3.5
B) 4.0
C) 4.5
D) 5.0

P4 Solution:

Bearing capacity factors for   35 and shape factors for square footing:

N q  41.44,N  42.43, S  0.85

Calculate the net bearing capacity in terms of “B”:

qnet   D f ( N q  1)  0.5 BN S


qnet  (125 lb3 )(3ft)(41.44  1)  0.5(125 lb3 )( B)(42.43)(0.85)
ft ft

qnet  15165 lb2  2254 lb3 B


ft ft

Calculate the allowable net bearing capacity in terms of “B”:

qnet 15165 ft 2  2254 ft3 B


lb lb
(qnet )all    7583 lb2  1127 lb3 B
FS 2 ft ft

The applied net bearing pressure equation in terms of “B” is given as:

Pnet Pnet Pnet


Qnet   2  B 2 
A B Qnet

XV
Refresher Class – Appendix C Geotechnical

The applied net bearing pressure must be less than or equal to the allowable
bearing capacity. To determine the minimum footing width, use the maximum
applied net bearing pressure, which is equal to the allowable bearing
capacity:

Qnet  (qnet )all  (Qnet )max  (qnet )all

Pnet Pnet 175, 000lb


 Bmin 2   
 Qnet max ( qnet ) all 7583 lb2  1127 lb3 B
ft ft

Simplify and solve the equation for “B”:

B 2 (7583  1127 B )  175,000


1127 B 3  7583B 2  175,000  0

Bmin  3.8 ft  Say 4 ft

The answer is (B)

TEST TIP:

For higher order polynomial equations, you can use trial and error to eliminate
answers, starting with one of the two middle values. For example, start with 4.0 ft:

1127(4.0)3  7583(4.0) 2  175,000  18,456


The positive number implies that 4.0 ft is greater than the minimum value of “B”.
Therefore choices C and D are definitely not the minimum value and can be
eliminated. Next check the next smallest value, 3.5 ft:

1127(3.5)3  7583(3.5) 2  175,000  33,788


The negative number implies that 3.5 ft is smaller than the minimum value of “B”,
which makes (B) the correct answer.

XVI
Refresher Class – Appendix C Geotechnical

P5 - Chapter 10
A 10-ft cantilever wall with the geometry shown is proposed. Both the backfill
and foundation sand have a moist unit weight of 130 pcf, saturated unit
weight of 138 pcf and internal angle of friction of 32 (corresponds to Ka =
0.307) . Assume  is equal to (2/3). It is expected that the groundwater table
could rise up to 3 ft above the base of the wall. Compute the factor of safety
against sliding and overturning for this condition. (Neglect passive pressure
and the weight of soil above the toe.)

P5 - Solution:
Calculate the active resultant force, Ra (lb/ft), acting on the plane A-A’, and
the sum of the moments, Mo (ft-lb/ft), to the base of the wall.

Ra  Ra1  Ra 2  Ra3  Ra 4
M o  Ra y  Ra1 y1  Ra 2 y 2  Ra 3 y3  Ra 4 y 4

XVII
Refresher Class – Appendix C Geotechnical

Moment Arm
Force (plf) Moment (ft-lb/ft)
(ft)
(130pcf )(7ft)2 (0.307) 7ft
 3ft  5.33
 979 979  5.33  5218
2 3
3ft
(130pcf )(7ft)(3ft)(0.307)  839  1.5 839  1.5  1259
2
(138pcf  62.4pcf )(3ft)2 (0.307) 3ft
1
 104 104  1  104
2 3
(62.4pcf )(3ft)2 3ft
1
 281 281  1  281
2 3
Ra  2203 M o  6862

Note: Reference the equations shown for “Case 3” to calculate R a and Mo.

Calculate the weight of the concrete wall and soil retained above the heel,
W (lb/ft), and the sum of the moments, MR (ft-lb/ft) to the toe of the wall.

W  W1  W2  W3  W4
M R  Wx  W1 x1  W 2 x 2  W3 x3  W 4 x 4

Force (plf) Moment Arm (ft) Moment (ft-lb/ft)


5ft
(130pcf)(5ft)(7ft)  4550  1.5ft  2.5ft  6.5 4550  6.5  29575
2
5ft
(138pcf)(5ft)(1.5ft)  1035  1.5ft  2.5ft  6.5 1035  6.5  6728
2
1.5ft
(150pcf )(1.5ft)(8.5ft)  1913  2.5ft  3.25 1913  3.25  6215
2
9ft
(150pcf)(9ft)(1.5ft)  2025  4.5 2025  4.5  9113
2
W  9523 M R  51631

XVIII
Refresher Class – Appendix C Geotechnical

Calculate the total resisting shear force, Fs (plf), developed along the base
of the wall.
2 
FS  W tan   (9523 plf ) tan   32    3719 plf
3 

Calculate the factor of safety against sliding:

 FR FS 3719 plf
FS SL     1.69
 FO Ra 2203 plf

Calculate the factor of safety against overturning.

 M R 51631 (ft  lb/ft)


FS OT    7.52
M O 6862 (ft  lb/ft)

P6 - Chapter 10
A cantilever wall with the geometry shown is proposed. Both the backfill and
foundation sand have a unit weight of 125 pcf, and internal angle of friction
of 30 (corresponds to Ka = 0.333). Assume  is equal to (2/3). If the factor
of safety against sliding must be at least 1.5, the minimum width of B is most
nearly:

A) 6
B) 9
C) 12
D) 15

XIX
Refresher Class – Appendix C Geotechnical

P6 - Solution:
Calculate the active resultant force, Ra (lb/ft).

lb 2
K a H 2 0.333(125 ft 3 )(12ft)
Ra    2997 lbft
2 2
Calculate the weight of the concrete wall and soil retained above the heel,
W (lb/ft).

W  W1  W2  W3

Force (plf)
1 (125pcf)(10.5ft)(B-3ft)  1312.5B  3937.5
2 (150pcf)(10.5ft)(1.5ft)  2362.5
3 (150pcf )(1.5ft)(B)  225B
 W  1537.5 B  1575.0

Calculate the total resisting shear force, Fs (plf), developed along the base
of the wall.
FS  W tan 
 (1537.5 B  1575.0 plf ) tan(20 )
 559.60 B plf  573.25 plf

Calculate the minimum width of B using the minimum factor of safety


against sliding:

 FR FS 559.60 B plf  573.25 plf


FS SL    1.5 
 FO Ra 2997 plf

(1.5  2997 plf )  573.25 plf


B  9.05 ft
559.60 psf

The answer is (B)

XX

You might also like